Sunteți pe pagina 1din 50

Ambulatory Care

Ambulatory Care
Ila M. Harris, Pharm.D., FCCP, BCPS
University of Minnesota
Minneapolis, Minnesota

ACCP Updates in Therapeutics® 2012: The Pharmacotherapy Preparatory Review and Recertification Course

1-231
Ambulatory Care

Learning Objectives: 3. Which one of the following types of measurement


best classifies the “number of times a short-acting
1. Select and monitor appropriate acute and preven- β2-agonist (SABA) is used in 1 month”?
tive treatment for pediatric and adult patients with A. Nominal.
asthma, adult patients with chronic obstructive B. Ordinal.
pulmonary disease, and conditions requiring anti- C. Interval.
coagulation, depending on patient-specific factors. D. Ratio.
2. Classify a patient according to asthma severity
class, and assess his or her control, according to 4. You are designing a study in which you will com-
the current National Institutes of Health National pare the percentage of patients with an asthma-
Heart, Lung and Blood Institute guidelines. related hospitalization receiving fluticasone/sal-
3. Discuss indications for warfarin and goal interna- meterol versus fluticasone alone. Which one of the
tional normalized ratio (INR) and therapy duration following statistical tests is best for analyzing this
for specific patients, and adjust therapy accord-
comparison?
ing to INR, other clinical findings, and/or patient
factors. A. ANOVA (analysis of variance).
4. Describe how to design a treatment plan for a pa- B. Chi-square.
tient receiving warfarin who needs to undergo an C. Mann-Whitney U test.
invasive procedure. D. Student unpaired t-test.
5. Determine the appropriate immunizations for an
adult given his or her age and medical conditions. 5. V.M. is a 22-year-old woman with asthma. Her
medications include albuterol (Ventolin hydro-
fluoroalkane [HFA]), metered dose inhaler (MDI)
Self-Assessment Questions: 2 puffs as needed, and fluticasone (Flovent) 110
Answers and explanations to these questions mcg/puff MDI 2 puffs 2 times/day. She received
may be found at the end of this chapter. the influenza vaccine during last year’s influenza
season, and her last tetanus vaccine (tetanus, diph-
1. K.R., a 20-year-old woman, is admitted to the hos- theria, and pertussis [Tdap]) was at age 17; there is
pital in acute respiratory distress. She states that no documentation of her receiving a pneumococcal
she has been using her boyfriend’s albuterol inhal- vaccine. Which one of the following vaccines best
er on a regular basis for the past 2 years. During represents which vaccine she should receive at her
the past few months, she has been using the inhaler next Family Medicine clinic appointment, sched-
throughout the day on a daily basis and sometimes uled in July?
at night. Which one of the following best classifies A. Influenza.
her asthma severity? B. Pneumococcal.
A. Mild intermittent. C. Td (tetanus and diphtheria).
B. Mild persistent. D. Herpes zoster.
C. Moderate persistent.
D. Severe persistent. 6. V.B. is a 70-year-old woman with a history
of recurrent deep venous thrombosis (DVT),
2. Which one of the following is the best maintenance hypertension, and hyperlipidemia. Her medications
therapy for K.R.’s asthma? include metoprolol succinate 100 mg/day, warfarin
A. Fluticasone (Flovent) low dose. 2.5 mg/day, hydrochlorothiazide 12.5 mg/day, and
B. Montelukast (Singulair). lovastatin 10 mg/day. Which one of the following
C. Fluticasone medium dose plus salmeterol best represents the goal international normalized
(Advair). ratio (INR) in recurrent DVT?
D. Theophylline. A. 1.8–2.5.
B. 2.0–3.0.

ACCP Updates in Therapeutics® 2012: The Pharmacotherapy Preparatory Review and Recertification Course

1-232
Ambulatory Care

C. 2.5–3.5.
D. 3.0–4.0.

7. S.T., a 30-year-old pregnant woman, had a DVT dur-
ing her pregnancy at 36 weeks. She received low-
molecular-weight heparin (LMWH) until delivery
(at 40 weeks). She has now delivered a healthy baby
boy, whom she is breastfeeding. Which one of the
following best represents how long S.T. should now
receive warfarin, after delivery?
A. Warfarin is contraindicated in women who
are breastfeeding; continue LMWH.
B. Warfarin should be used for 6 weeks.
C. Warfarin should be used for 3 months.
D. Warfarin should be used for the rest of her
life.

8. A 60-year-old man with mild chronic obstructive
pulmonary disease (COPD) has been using
albuterol HFA (ProAir) 2 puffs 4 times/day as
needed. His symptoms have worsened during the past
few months, and now, he has persistent symptoms
and shortness of breath, even while walking around
his one-level house. His Modified British Medical
Research Council (mMRC) score is 2. His spirometry
showed a forced expiratory volume in 1 second
(FEV1) of 70% of predicted and an FEV1/forced vital
capacity (FEV1/FVC) of 60% of predicted. He has
never had a COPD exacerbation. Which one of the
following medications is best to initiate?
A. Fluticasone (Flovent).
B. Tiotropium (Spiriva).
C. Montelukast (Singulair).
D. Omalizumab (Xolair).

ACCP Updates in Therapeutics® 2012: The Pharmacotherapy Preparatory Review and Recertification Course

1-233
Ambulatory Care

I. ASTHMA

Guidelines:
National Institutes of Health (NIH) National Heart Lung and Blood Institute (NHLBI). National Asthma
Education and Prevention Program Guidelines (NAEPP). NAEPP Expert Panel Report 3. NIH Publication
08-5846. July 2007. Available at www.nhlbi.nih.gov/guidelines/asthma/. Accessed December 6, 2011.

A. Definition: Asthma is a chronic inflammatory disorder of the airways causing recurrent episodes of
wheezing, breathlessness, cough, and chest tightness, particularly at night or early in the morning. During
episodes, there is variable airway obstruction, often reversible spontaneously or with treatment. There is
also increased bronchial hyperresponsiveness to a variety of stimuli.

B. Diagnosis
1. Episodic symptoms of airflow obstruction are present.
2. Airway obstruction is reversible (FEV1 improves by 12% or more after SABAs).
3. Alternative diagnoses are excluded. Asthma versus COPD:
a. Cough is usually nonproductive with asthma and productive with COPD.
b. FEV1 is reversible with asthma but not reversible with COPD.
c. Cough is worse at night and early in the morning with asthma; throughout the day with COPD
d. Asthma is often related to allergies/environmental triggers; COPD has a common history of
smoking.
e. Asthma can be reversible; lung damage from COPD is irreversible.
4. Exercise-induced bronchospasm
a. Presents with cough, shortness of breath, chest pain or tightness, wheezing, or endurance problems
during exercise
b. Diagnosis is made by an exercise challenge, in which a 15% decrease in FEV1 or peak expiratory
flow is seen before and after exercise, measured at 5-minute intervals for 20–30 minutes.
c. Prevention and treatment
i. Long-term control therapy, if appropriate (initiate or step up)
ii. Pretreatment with a SABA before exercise
iii. LTMs can attenuate symptoms in 50% of patients.

C. Classification of Asthma Severity and Control

ACCP Updates in Therapeutics® 2012: The Pharmacotherapy Preparatory Review and Recertification Course

1-234
Ambulatory Care

Table 1. Classification of Asthma Severity in Adults and Childrena


Age Group
Components (years old) Intermittent Mild Persistent Moderate Persistent Severe Persistent
Frequency of All ages ≤ 2 days/week > 2 days/week, Daily Throughout the day
symptoms but not daily
Nighttime ≥ 12 ≤ 2 times/month 3 or 4 times/month More than once weekly, Often 7 times/week
awakening 5–11 but not nightly

0–4 0 1 or 2 times/month 3 or 4 times/month More than once


weekly
SABA; used for ≤ 2 days/week > 2 days/week, Daily Several times a day
symptom control All ages but not daily

Interference with All ages None Minor limitation Some limitations Extremely limited
normal activity
FEV1/FVCb ≥ 12 Normal Normal Reduced 5% Reduced > 5%
5–11 > 85% > 80% 75%–80% < 75%
0–4 N/A
FEV1 ≥ 12 > 80% > 80% > 60% to < 80% < 60%
(% of normal) 5–11 (normal) (normal)
0–4 N/A
Exacerbations ≥ 12 0 or 1/year ≥ 2/year ≥ 2/year ≥ 2/year
requiring oral 5–11
steroids 0–4 0 or 1/year ≥ 2 in 6 months or ≥ 4 wheezing episodes per yearc
Recommended ≥ 12 Step 1 Step 2 Step 3d and consider Step 4 or 5 and
step for initiating short course of consider short
treatment (see oral steroids course of oral
Table 4) steroids
5–11 Step 3d or 4 and
consider short
course of oral
steroids
0–4 Step 3 and consider
short course of
oral steroids
a
The patient is classified according to the sign or symptom that is in the most severe category.
b
Normal FEV1/FVC: 8–19 years old: 85%, 20–39 years old: 80%, 40–59 years old: 75%, 60–80 years old: 70%.
c
Episodes lasting more than 1 day and risk factors for persistent asthma
d
For age 5–11, initial step 3 therapy should be medium-dose ICS.
FEV1 = forced expiratory volume in 1 second; FVC = forced vital capacity; ICS = inhaled corticosteroid; N/A = not applicable.
Adapted from NIH asthma guidelines. National Institutes of Health National Heart, Lung and Blood Institute. National Asthma Education and
Prevention Program (NAEPP) guidelines. NAEPP Expert Panel Report 3. NIH Publication 08-5846. Available at www.nhlbi.nih.gov/guidelines/index.
htm. Accessed September 23, 2011.

ACCP Updates in Therapeutics® 2012: The Pharmacotherapy Preparatory Review and Recertification Course

1-235
Ambulatory Care

Patient Case
1. B.W. is a 20-year-old woman who attends the university. She joined a fitness club to lose weight; she has
never exercised before. She comes to the clinic today because she coughs and has some trouble breathing in
the new aerobics class that she attends twice weekly; however, it does not limit her activity. Which one of the
following best classifies B.W.’s asthma?
A. Intermittent.
B. Mild persistent.
C. Moderate persistent.
D. Severe persistent.

Table 2. Interpreting Spirometry


Component What It Measures Normal Values
FEV1 Volume of air exhaled forcefully in the Normal is 80% or greater
first second of maximal expiration In asthma, reversibility is shown by an
increase of FEV1 by ≥ 12% after SABA
FVC The maximum volume of air that can Reported in liters and % predicted
be exhaled after full inspiration Normal adults can empty 80% of air in < 6 seconds
FEV1/FVC Differentiates between obstructive Normal: Within 5% of predicted range, which
ratio and restrictive disease varies with age; usually 75%–80% in adults
Decreased in obstructive disease (asthma, COPD) (< 70%)
Normal/high in restrictive disease (pulmonary fibrosis)
COPD = chronic obstructive pulmonary disease; FEV1/FVC = forced expiratory volume/forced vital capacity; SABA = short-acting β-agonist.

ACCP Updates in Therapeutics® 2012: The Pharmacotherapy Preparatory Review and Recertification Course

1-236
Ambulatory Care

Table 3. Assessing Asthma Control in Adults and Children


Age Group
Component (years old) Well Controlled Not Well Controlled Very Poorly Controlled
Symptoms ≥ 12 ≤ 2 days/week > 2 days/week
5–11 ≤ 2 days/week but > 2 days/week or > 1x/ Throughout the day
0–4 not > 1x each day day on any day
Nighttime ≥ 12 ≤ 2 times/month 1–3 times/week ≥ 4 times/week
awakenings 5–11 ≤ 1 time/month ≥ 2 times/month ≥ 2 times/week
0–4 > 1 time/month > 1 time/week
Interference with All ages None Some limitation Extremely limited
normal activity
Short-acting β2-
agonist use for All ages ≤ 2 days/week > 2 days/week Several times a day
symptom controla
FEV1 or peak flow ≥ 12 > 80% of predicted/ 60%–80% of predicted/ < 60% of predicted/
5–11 personal best personal best personal best
0–4 N/A N/A N/A
Questionnaires ≥ 12
ATAQ (N/A if < 12) 0 1 or 2 3 or 4
ACQ ≤ 0.75 ≥ 1.5 N/A
ACT ≥ 20 16–19 ≤ 15
Exacerbations ≥ 12
requiring oral 5–11 0 or 1/year ≥ 2/year ≥ 2/year
steroids 0–4 2 or 3 times/year > 3 times/year
Recommended Maintain current step; Consider short course
action for All ages regular follow-up Step-up 1 step of oral steroids
treatment every 1–6 months; Reevaluate in 2–6 weeks Step-up 1 or 2 steps
consider step-down Reevaluate in 2 weeks
if well controlled
≥ 3 months
a
Does not include β2-agonist used to prevent exercise-induced asthma.
ACQ = Asthma Control Questionnaire; ACT = Asthma Control Test; ATAQ = Asthma Therapy Assessment Questionnaire; FEV1 = forced
expiratory volume in 1 second; N/A = not applicable.
Adapted from NIH asthma guidelines. National Institutes of Health National Heart, Lung and Blood Institute. National Asthma Education and
Prevention Program (NAEPP) guidelines. NAEPP Expert Panel Report 3. NIH Publication 08-5846. Available at www.nhlbi.nih.gov/guidelines/
index.htm. Accessed September 23, 2011.

D. Treatment Goals
1. Minimal or no chronic symptoms day or night
2. Minimal or no exacerbations
3. No limitations on activities; no school/work missed
4. Maintain (near) normal pulmonary function.
5. Minimal use of SABAs
6. Minimal or no adverse effects from medications

ACCP Updates in Therapeutics® 2012: The Pharmacotherapy Preparatory Review and Recertification Course

1-237
Ambulatory Care

E. Treatment Guidelines

Table 4. Treatment Guidelines


Age Group
Step (years old) Long-term Control Quick Relief
1 All ages No controller needed Use SABA PRN
SABA > 2 times/
2 ≥ 12 Preferred: Low-dose ICS week (excluding
5–11 • Alternatives: LTM, theophylline, or cromolyna preexercise doses)
0–4 Preferred: Low-dose ICS indicates inadequate
Alternatives: Montelukast or cromolyna control and need to
step-up treatment
3 ≥ 12 • Preferred: Low-dose ICS plus LABA
OR medium-dose ICS alone
• Alternative: Low-dose ICS plus LTM or theophylline
5–11 Preferred: Low-dose ICS plus LABA, LTM, or
theophylline OR medium-dose ICS alone (medium-
dose ICS preferred as initial therapy)
0–4 Medium-dose ICS
4 ≥ 12 • Preferred: Medium-dose ICS plus LABA
5–11 • Alternative: Medium-dose ICS plus LTM or theophylline
0–4 Preferred: Medium-dose ICS plus LABA or montelukast
Alternative: Medium-dose ICS plus other LTM or theophylline
5 ≥ 12 • High-dose ICS plus LABA AND consider
omalizumab for patients with allergic asthma
5–11 Preferred: High-dose ICS plus LABA
Alternative: High-dose ICS plus LTM or theophylline
0–4 High-dose ICS plus LABA or montelukast
6 ≥ 12 • High-dose ICS plus LABA plus systemic corticosteroids AND
consider omalizumab for patients with allergic asthma
5–11 Preferred: High-dose ICS plus LABA
plus systemic corticosteroids
Alternative: High-dose ICS plus LTM or
theophylline plus systemic corticosteroids
0–4 High-dose ICS plus LABA or montelukast
plus systemic corticosteroids
a
Cromolyn and nedocromil are included in the National Asthma Education and Prevention Program guidelines. Cromolyn and nedocromil
inhalers have been discontinued by the manufacturer; only generic cromolyn nebulization solution is still available.
ICS = inhaled corticosteroid; LABA = long-acting β2-agonist; SABA = short-acting β2-agonist; LTM = leukotriene modifier; PRN = as needed;
SR = sustained release.
Adapted from NIH asthma guidelines. National Institutes of Health National Heart, Lung and Blood Institute. National Asthma Education and
Prevention Program (NAEPP) guidelines. NAEPP Expert Panel Report 3. NIH Publication 08-5846. Available at www.nhlbi.nih.gov/guidelines/
index.htm. Accessed September 23, 2011.

ACCP Updates in Therapeutics® 2012: The Pharmacotherapy Preparatory Review and Recertification Course

1-238
Ambulatory Care

Patient Cases
2. Which one of the following medications is the best option for B.W., from case 1, in addition to albuterol MDI
1 or 2 puffs before exercise?
A. Albuterol MDI 1 or 2 puffs 4 times/day as needed.
B. Montelukast 10 mg/day.
C. Omalizumab 150 mg subcutaneously every 4 weeks.
D. Mometasone MDI 220 mcg/puff 1 puff/day.

3. Your recommendation has slightly improved her symptoms. However, she has now begun coughing at night
once weekly. Which one of the following is the most appropriate treatment?
A. Budesonide-formoterol MDI 80/4.5 2 puffs 2 times/day.
B. Montelukast 10 mg/day.
C. Salmeterol MDI 2 puffs 2 times/day.
D. Fluticasone 110 mcg/puff 1 puff 2 times/day.

4. B.W. returns 1 month later. She is no longer awakening at night. She uses her albuterol inhaler once weekly
to treat her symptoms. She also uses her albuterol inhaler 2 puffs 5 days/week before she works out at the
gym. She does not have symptoms while working out. Which one of the following is the most appropriate
action to take at this time?
A. No medication change is needed.
B. Increase fluticasone to 110 mcg/puff 2 puffs 2 times/day.
C. Add formoterol inhale 1 capsule 2 times/day.
D. Add montelukast 10 mg/day.

ACCP Updates in Therapeutics® 2012: The Pharmacotherapy Preparatory Review and Recertification Course

1-239
Ambulatory Care

F. Pharmacologic Therapy for Asthma

Table 5. Pharmacologic Agent Use for Asthma and COPDa


Generic Brand Dose Adverse Effects Comments
Corticosteroid inhalers
Beclomethasone MDI QVAR (HFA) See ICS dosing INHALED ICS are first line for persistent asthma
40 mcg/puff table Oral candidiasis • Use HOLDING CHAMBERS only
80 mcg/puff Hoarseness if needed for technique; not needed
Fluticasone MDI Flovent HFA May slow bone growth in or well studied with HFA inhalers;
44 mcg/puff children but similar holding chambers are only for MDIs
110 mcg/puff adult height – cannot be used for DPIs; holding
220 mcg/puff chambers with a mask can be used
Fluticasone DPI Flovent SYSTEMIC for young children
50 mcg/puff Diskus Cushing effects • RINSE MOUTH with water after
100 mcg/puff Growth retardation inhalations
250 mcg/puff Osteoporosis • Use corticosteroid inhaler as
Hypertension SCHEDULED, not as needed
Mometasone DPI Asmanex Cataracts • Onset of improvement is 5–7 days.
220 mcg/puff Twisthaler Glucose intolerance Additional benefit may be seen over
Budesonide DPI Pulmicort Skin thinning several weeks
90 mcg/dose Flexhaler and Myopathy • Consider calcium and vitamin D
180 mcg/dose Respules Euphoria supplements in adults, particularly in
0.25-, 0.5-, and Depression perimenopausal women
1-mg/2-mL nebs • Pulmicort Respules are only
Ciclesonide MDI Alvesco nebulized steroid available
80 mcg/puff (HFA)
160 mcg/puff
Anticholinergics
Ipratropium MDI Atrovent HFA 2–4 puffs TID– Headache Used mainly for COPD or for acute
17 mcg/puff QID (up to 12 Flushed skin asthma exacerbations requiring
puffs/24 hours) Blurred vision emergency treatment
Tachycardia Duration: 2–8 hours
Palpitations Also available as a solution for
nebulization
Tiotropium DPI Spiriva Inhale 1 Used for COPD; not currently
18 mcg capsule/day recommended for asthma.
Long acting; not for rapid relief
Duration: > 24 hours
β2-Agonists (short acting) – SABA
Albuterol MDI Proventil 2 puffs every Tremor Used for acute bronchospasm; regular
90 mcg/puff HFA 4–6 hours PRN Tachycardia, use indicates poor control
Ventolin HFA Hypokalemia - Also available as solution for
ProAir – HFA Hypomagnesemia nebulization
Hyperglycemia - Duration of effect (MDI): 3–4 hours
Tachyphylaxis (up to 6)
Levalbuterol MDI Xopenex 2 puffs every - R-enantiomer of albuterol
45 mcg/puff HFA 4–6 hours PRN - Also available as a solution for
nebulization
- Duration (MDI): 3–4 hours (up to 6)
Pirbuterolb Maxair 2 puffs every Breath-actuated MDI
200 mcg/puff Autohalerb 4–6 hours PRN - Duration: 5 hours

β2-Agonists (long acting) – LABA


Salmeterol DPI Serevent Inhale 1 blister/ Tremor Not for acute symptoms
50 mcg/puff Diskus puff BID Tachycardia Should NOT be used as monotherapy for
Electrolyte effects rare asthma
Duration: 8–12 hours

ACCP Updates in Therapeutics® 2012: The Pharmacotherapy Preparatory Review and Recertification Course

1-240
Ambulatory Care

Table 5. Pharmacologic Agent Use for Asthma and COPDa (continued)


Generic Brand Dose Adverse Effects Comments
Formoterol DPI Foradil Inhale 1 capsule Onset of action 1–3 minutes, but should not
12-mcg capsule Aerolizer BID be used as acute therapy
Should NOT be used as monotherapy
Formoterol Perforomist 20-mcg BID for asthma
20-mcg/2-mL nebs nebs Duration of MDI: 8–12 hours
Formoterol Aerolizer is indicated to prevent
Arformoterol Brovana 15-mcg BID exercise-induced bronchospasm; should be
15-mcg/2-mL nebs nebs used at least 15 minutes before exercise

Arformoterol is the R,R-isomer of racemic


formoterol
Indacaterol inhalation Arcapta Inhale 1 capsule Indacaterol is only indicated for COPD
powder Neohaler once daily NOT indicated for use in asthma at all
75-mcg capsule Approved by FDA July 2011
Duration of action: 24 hours
Combination inhalers
Albuterol 103 mcg/ Combiventb 2 puffs QID Primarily used for COPD
puff plus Combivent Respimat (newly approved
ipratropium propellant-free mist inhaler) will be
18 mcg/puff MDIb available starting mid-2012
- 100 mcg of albuterol/20 mcg of
ipratropium
Combination solution for nebulization is
also available as DuoNeb or generic
Fluticasone – Advair Diskus 1 puff BID Combination of ICS and LABA
salmeterol DPI
100/50, 250/50,
500/50 mcg/puff
Fluticasone – Advair HFA 2 puffs BID
salmeterol MDI
45/21, 115/21, 230/21
mcg/puff
Budesonide – Symbicort 2 puffs BID
formoterol MDI (HFA)
80/4.5, 160/4.5 mcg/
puff
Mometasone – Dulera 2 puffs BID
formoterol MDI (HFA)
100/5, 200/5 mcg/puff
Methylxanthine
Theophylline Theo-Dur 10 mg/kg/day At high levels: Achieve concentrations of 5–15 mcg/mL
Uniphyl (IBW) – Nausea Beneficial for night symptoms
Liquids, capsules, Theo-24 Divided Vomiting Not for acute relief
sustained-release according to CNS stimulation Duration: variable; up to 24 hours
capsules (many dosage formulation Headache
strengths) - Adjust Tachycardia, SVT
according to Seizures
concentration Hematemesis
Max: 16 mg/kg/ Hyperglycemia
day (children < Hypokalemia
12 years); 800 At usual levels:
mg/day (adults) Insomnia
GI upset
Smokers may Increased
need higher hyperactivity in some
doses at more children
frequent Difficult urination in
intervals BPH

ACCP Updates in Therapeutics® 2012: The Pharmacotherapy Preparatory Review and Recertification Course

1-241
Ambulatory Care

Table 5. Pharmacologic Agent Use for Asthma and COPDa (continued)


Generic Brand Dose Adverse Effects Comments
Leukotriene modifiers (note: *FDA caution)
Zafirlukast Accolate 20 mg BID Hepatotoxicity: Drug interactions: Warfarin, erythromycin,
10-mg tablet Monitor LFTs theophylline
20-mg tablet (baseline, every For ≥ 5 years
month × 3 months, Bioavailability decreases with food; take 1
every 2–3 months for hour before or 2 hours after meals
Montelukast Singulair 5–10 mg/day 1 year for montelukast Drug interactions: Phenobarbital
Oral 10-mg tablet and zafirlukast) FDA approved for use in ≥ 1 year; used in
Chewable 4- and 5-mg Headache 6 months and older
tablets GI upset Churg-Strauss syndrome associated with
Oral granules 4-mg/ tapering doses of steroids
packet *Risk of
Zileuton Zyflo CR 1200 mg BID neuropsychiatric Drug interactions: Warfarin and
600-mg CR tablet events (behavior theophylline
and mood changes: Only for those 12 years and older
aggression, agitation,
anxiousness, dream
abnormalities,
hallucinations,
depression,
insomnia, irritability,
restlessness, suicidal
thinking and behavior,
tremor)

Monoclonal antibody
Omalizumab Xolair 150–375 mg SC Injection site reactions MOA: Inhibits IgE binding to high-affinity
every 2–4 weeks Urticaria IgE receptors on mast cells and basophils
Dose and Thrombocytopenia Indicated in moderate to severe persistent
frequency based (transient) allergy-related asthma
on baseline IgE Anaphylaxis (rare) Use in ≥ 12 years old
and weight in Malignancy Half-life: 26 days
kilograms Second-line therapy
Do not inject Very expensive
> 150 mg per
injection site
a
The following MDIs have been discontinued by the manufacturers and are no longer available: AeroBid (flunisolide) MDI (June 2011), Azmacort
(triamcinolone) MDI (December 2009), Intal (cromolyn) MDI (August 2009), Intal (cromolyn) nebulization solution (June 2008), and Tilade
(nedocromil) MDI (April 2008). They are not included in this table. OTC Primatene Mist has also been discontinued and is no longer available as
of 12/31/2011.
b
These inhalers still contain CFCs and are being phased out by the manufacturers. They may no longer be sold after the following dates: Maxair
(pirbuterol) (12/31/2013) and Combivent (albuterol/ipratropium) (12/31/2013).
BID = 2 times/day; CNS = central nervous system; COPD = chronic obstructive pulmonary disease; CR = controlled release; DOC = drug of
choice; DPI = dry powder inhaler; FDA = U.S. Food and Drug Administration; GERD = gastroesophageal reflux disease; GI = gastrointestinal;
HFA = hydrofluoroalkane; ICS = inhaled corticosteroid; IgE = immunoglobulin E; LABA = long-acting β2-agonist; LFT = liver function test;
MDI = metered dose inhaler; MOA = mechanism of action; nebs = nebulizers; PRN = as needed; QID = 4 times/day; SC = subcutaneously; SVT
= supraventricular tachycardia; TID = 3 times/day.

ACCP Updates in Therapeutics® 2012: The Pharmacotherapy Preparatory Review and Recertification Course

1-242
Ambulatory Care

Table 6. Inhaled Corticosteroid Daily Dosing in Children and Adultsa


Low Dose (mcg/day) Medium Dose (mcg/day) High Dose (mcg/day)
Inhaled Corticosteroids Steps 2 and 3 Steps 3 and 4 Steps 5 and 6
Age Group (years) 0–4 5–11 ≥ 12 0–4 5–11 ≥ 12 0–4 5–11 ≥ 12
Budesonide N/A 180–400 180–600 N/A > 400–800 > 600–1200 N/A > 800 > 1200
(Pulmicort DPI 90, 180)
Fluticasone
Flovent HFA 44, 110, 220 176 88–176 88–264 > 176–352 > 176–352 > 264–440 > 352 > 352 > 440
Flovent DPI 50, 100, 250 N/A 100–200 100–300 N/A > 200–400 > 300–500 N/A > 400 > 500
Beclomethasone N/A 80–160 80–240 N/A > 160–320 > 240–480 N/A > 320 > 480
(QVAR HFA 40, 80)
Mometasone (Asmanex 110 110 200 110 110 400 110 110 > 400
DPI 110, 220): delivers (age 4 only) (age 4 only) (age 4 only)
100 and 200 mcg/puff)b
Ciclesonidec N/A N/A 160 N/A N/A 320 N/A N/A 640
(Alvesco HFA 80, 160)
Budesonide suspension 0.25–0.5 mg 0.5 mg N/A > 0.5–1 mg 1 mg N/A > 1 mg 2 mg N/A
for nebulization
a
Effective December 31, 2009: Azmacort (triamcinolone) MDI (12/31/2009) and AeroBid (flunisolide) MDI (6/30/2011) have been discontinued by
the manufacturers and are no longer available.
b
The guidelines state the delivered dose of mometasone, not the actual dose; indicated in ages 4–11 after guidelines were published; doses are
estimated from package insert.
c
Ciclesonide was not available when the National Asthma Education and Prevention Program guidelines were published. The dose ranges are
estimated from the package insert.
CFC = chlorofluorocarbon; DPI = dry powder inhaler; HFA = hydrofluoroalkane; MDI = metered dose inhaler.
Adapted from NIH asthma guidelines. National Institutes of Health National Heart, Lung and Blood Institute. National Asthma Education and
Prevention Program (NAEPP) guidelines. NAEPP Expert Panel Report 3. NIH Publication 08-5846. Available at www.nhlbi.nih.gov/guidelines/
index.htm. Accessed September 23, 2011.

Patient Cases
5. D.B. is a 9-year-old boy who has asthma symptoms once or twice a week. He is awakened twice weekly at night
with coughing and trouble breathing. Which one of the following best classifies his asthma severity?
A. Intermittent.
B. Mild persistent.
C. Moderate persistent.
D. Severe persistent.

6. In addition to albuterol MDI 1–2 puffs every 4–6 hours as needed, which one of the following is best for D.B.’s
initial therapy?
A. Beclomethasone 80 mcg/puff 1 puff 2 times/day.
B. Montelukast 10 mg/day.
C. Fluticasone/salmeterol 100/50 mcg 1 puffs 2 times/day.
D. Fluticasone 110 mcg/puff 1 puff 2 times/day.

G. Long-Acting β2-Agonists (LABAs): The U.S. Food and Drug Administration (FDA) issued a safety
announcement in June 2010 because of safety concerns with LABAs.
1. Use of a LABA alone without a long-term asthma control drug, such as an inhaled corticosteroid (ICS), is
contraindicated.
2. LABAs should not be used in patients whose asthma is adequately controlled on low- or medium-dose ICSs.

ACCP Updates in Therapeutics® 2012: The Pharmacotherapy Preparatory Review and Recertification Course

1-243
Ambulatory Care

3. LABAs should only be used as additional therapy for patients who are currently taking, but not
adequately controlled on, a long-term asthma control agent, such as an ICS.
4. Once asthma control is achieved and maintained, patients should be assessed at regular intervals and
step down (e.g., discontinue the LABA), if possible, and the patient should continue to be treated with
a long-term asthma control agent, such as an ICS.
5. Pediatric and adolescent patients who require a LABA and an ICS should use a combination product to
ensure adherence with both medications.

H. New Data in Asthma Treatment


(not included in the current guidelines; more data needed before using in clinical practice)
1. Tiotropium for persistent asthma
a. A recent study evaluated adding tiotropium versus adding LABA (salmeterol) versus doubling the
ICS dose in patients uncontrolled on low-dose beclomethasone (Peters SP, et al. TALC study. N
Engl J Med 2010;363:1715–26).
b. Results: Adding tiotropium resulted in significantly greater improvements in peak expiratory flow,
FEV1, symptom control, and number of asthma control days than doubling the ICS dose.
c. Adding tiotropium was at least non-inferior to adding a LABA for all outcomes studied, and it
increased pre-bronchodilator FEV1 more than did salmeterol (p=0.003).
d. Conclusion: Adding tiotropium to low-dose ICS is an option if asthma is uncontrolled; non-
inferior to adding LABA and superior to doubling ICS
2. ICS/SABA for rescue treatment in children
a. Randomized, double-blind, placebo-controlled trial in children/adolescents with mild persistent
asthma randomized to four different groups (TREXA study; Lancet 2011;377:650–7):

Group Controller Therapy Rescue Therapy


Combined 40 mcg of BID BMS BMS (80 mcg) + albuterol (2 puffs)
Daily BMS 40 mcg of BID BMS Albuterol alone
Rescue BMS Placebo BMS (80 mcg) + albuterol
Placebo Placebo Albuterol
BID = twice daily; BMS = beclomethasone.

b. Results: Rescue beclomethasone (BMS) compared with placebo resulted in significantly lower
frequency of treatment failure (8.5% vs. 23%; p=0.024).
c. Conclusion: In mild persistent asthma in children, if step-down therapy is needed, rescue BMS and
albuterol may be preferred over rescue albuterol alone.
3. Leukotriene modifiers (LTMs) versus ICS as first-line controller
a. Parallel, multicenter, pragmatic trial in the United Kingdom to evaluate “real-world” effectiveness
(N Engl J Med 2011;364:1695–70)
b. LTM: Mostly montelukast; ICS: mostly BMS
c. Results: For most outcomes (Asthma Control Questionnaire [ACQ] score, asthma exacerbations),
no significant difference between LTM and ICS as first-line controller therapy; mini-AQLQ
(Asthma Quality of Life Questionnaire) scores did not all meet equivalence; adherence was higher
with LTM (65%) versus ICS (41%) [NS]
d. Conclusion: In a real-world setting, LTMs may be similar to ICS in efficacy for first-line controller
therapy.
4. LTM versus LABA as add-on therapy
a. Parallel, multicenter, pragmatic trial in the United Kingdom to evaluate “real-world”
effectiveness of adding on LTM versus LABA in patients already receiving ICS (N Engl J Med
2011;364:1695–707)
b. LTM: Mostly montelukast; LABA: mostly salmeterol

ACCP Updates in Therapeutics® 2012: The Pharmacotherapy Preparatory Review and Recertification Course

1-244
Ambulatory Care

c. Results: For most outcomes (ACQ score, asthma exacerbations), no significant difference between
LTM and LABA as add-on controller therapy; adherence was significantly higher with LTM (74%)
versus LABA (46%). However, only 13% in LABA group used combination ICS-LABA inhalers;
the rest were given 2 separate inhalers. This could have affected adherence.
d. Conclusion: In a real-world setting, LTMs may be equivalent to LABA as add-on therapy in
patients already receiving ICS.

I. Asthma Action Plan


1. Usually symptom based (equal benefits of symptom-based or peak flow–based monitoring); home
treatment of an asthma exacerbation
2. Green zone
a. Doing well; no/minimal symptoms of coughing, wheezing, or dyspnea (or peak expiratory flow
rate [PEFR] 80%–100% of personal best)
b. Take long-term asthma control agent only (if one is prescribed).
c. Use 2 puffs of SABA 5–15 minutes before exercise if exercise-induced asthma and before known
triggers. Use also as needed for symptoms.
3. Yellow zone
a. Getting worse; increased frequency of symptoms (e.g., coughing, wheezing, and/or dyspnea) (or
PEFR 50%–79% of personal best)
b. Use SABA: 2–4 puffs by MDI or 1 nebulizer treatment; may repeat in 20 minutes if needed. May
be as many as 6 puffs if needed. Reassess 1 hour after initial treatment.
i. If complete response at 1 hour, contact clinician for follow-up instructions and consider oral
corticosteroid (OCS) burst.*
ii. If incomplete response in 1 hour (still some coughing, wheezing, and/or dyspnea), repeat
SABA and add OCS burst*; contact clinician that day for further instructions
iii. If poor response in 1 hour (marked coughing, wheezing, and/or dyspnea), repeat SABA
immediately; add OCS burst*; contact clinician immediately, and proceed to the emergency
department (ED) if the distress is severe and unresponsive to treatment; consider calling 911
c. May continue to use SABA every 3–4 hours regularly for 24–48 hours
4. Red zone
a. Medical alert; marked coughing, wheezing, and/or dyspnea; inability to speak more than short
phrases; use of accessory respiratory muscles; drowsiness (or PEFR less than 50% of personal best)
b. Begin treatment and consult clinician immediately.
c. Use SABA: 2–6 puffs by MDI or 1 nebulizer treatment; repeat every 20 minutes up to 3 times; add
OCS burst.* Higher dose of 4–6 puffs SABA MDI is usually recommended.
d. If incomplete or poor response, repeat SABA immediately; proceed to the ED or call 911 if
distress is severe and unresponsive to treatment
e. Call 911 or go to the ED immediately if lips or fingernails are blue or gray or if there is trouble
walking or talking because of shortness of breath.
f. Continue to use SABA every 3–4 hours regularly for 24–48 hours.

*Oral prednisone burst; 40–60 mg/day for 5–10 days (adults) or 1–2 mg/kg/day (maximum 60 mg/day)
for 3–10 days (children).

5. After initial treatment, immediate medical attention is required if patient is at high risk of a fatal attack.
Risk factors:
a. Asthma: History of severe attack (previous intubation or intensive care unit admission for asthma),
two or more asthma hospitalizations for asthma in past year, three or more ED visits for asthma in
past year, hospitalization or ED visit for asthma in past month, using more than two canisters of
SABA a month, difficulty perceiving asthma symptoms
b. Social: Low socioeconomic status or inner-city residence, illicit drug use, major psychosocial problems
c. Comorbidities: Cardiovascular disease, other chronic lung disease, chronic psychiatric disease

ACCP Updates in Therapeutics® 2012: The Pharmacotherapy Preparatory Review and Recertification Course

1-245
Ambulatory Care

Patient Cases
7. J.H. is a 25-year-old woman using Advair (fluticasone/salmeterol) 250/50 1 puff 2 times/day and albuterol
HFA 1–2 puffs every 4–6 hours as needed. You are developing an asthma action plan for her. Which one of
the following best represents her green zone instructions?
A. Discontinue fluticasone/salmeterol because her asthma is under good control.
B. Use fluticasone/salmeterol regularly; may use albuterol HFA 1–2 puffs every 4–6 hours if needed.
C. Use albuterol HFA 2 puffs; repeat in 20 minutes if needed, and then reassess.
D. Use albuterol HFA 6 puffs; repeat in 20 minutes; start prednisone 50 mg once daily for 5 days, and
then reassess.

8. Which one of the following, in addition to using her fluticasone/salmeterol regularly, best represents J.H.’s
yellow zone instructions (mild exacerbation) for home treatment?
A. Albuterol HFA 1–2 puffs every 4–6 hours if needed.
B. Albuterol HFA 2 puffs; repeat in 20 minutes if needed, and then reassess.
C. Albuterol HFA 6 puffs; repeat in 20 minutes; start prednisone 50 mg once daily for 5 days, and then
reassess.
D. Albuterol HFA 10 puffs; repeat every 20 minutes for 4 hours; start prednisone 50 mg once daily for 5
days, and then reassess.

9. Which one of the following, in addition to her use of fluticasone/salmeterol regularly, best represents J.H.’s
red zone instructions for home treatment?
A. May use albuterol HFA 1–2 puffs every 4–6 hours if needed.
B. Albuterol HFA 2 puffs; repeat in 20 minutes if needed; then reassess.
C. Albuterol HFA 6 puffs; repeat in 20 minutes; start prednisone 50 mg once daily for 5 days; then
reassess.
D. Albuterol HFA 10 puffs; repeat every 20 minutes for 4 hours; start prednisone 50 mg once daily for 5
days; then reassess.

J. Managing Exacerbations: Initial – ED or Hospital

ACCP Updates in Therapeutics® 2012: The Pharmacotherapy Preparatory Review and Recertification Course

1-246
Ambulatory Care

Table 7. Classifying Severity of Asthma Exacerbations in the Urgent or Emergency Care Settinga
Symptoms and Signs Initial PEF or FEV1b Clinical Course
Mild Dyspnea only with activity ≥ 70% of predicted Usually cared for at home
or personal best Prompt relief with an inhaled SABA
Possible short course of OCS
Moderate Dyspnea interferes with 40%–69% of predicted Usually requires office or ED visit
or limits usual activity or personal best Relief from frequent inhaled SABAs
OCS; some symptoms last for 1–2
days after treatment is begun
Severe Dyspnea at rest; interferes < 40% of predicted Usually requires ED visit and
with conversation or personal best likely hospitalization
Partial relief from frequent inhaled SABA
Oral systemic corticosteroids;
some symptoms last for > 3 days
after treatment is begun
Adjunctive therapies are helpful
Subset: Life Too dyspneic to < 25% of predicted Requires ED/hospitalization; possible ICU
threatening speak; perspiring or personal best Minimal or no relief from
frequent inhaled SABAs
IV corticosteroids
Adjunctive therapies are helpful
a
For all ages.
b
Lung function measures (PEF or FEV1) may be useful for children ≥ 5 years but may not be obtainable in children during an exacerbation.
ED = emergency department; FEV1 = forced expiratory volume in 1 second; ICU = intensive care unit; IV = intravenous; OCS = oral
corticosteroid; PEF = peak expiratory flow; SABA = short-acting ß2-agonist.
Adapted from NIH asthma guidelines. National Institutes of Health National Heart, Lung and Blood Institute. National Asthma Education
and Prevention Program Guidelines (NAEPP). NAEPP Expert Panel Report 3. NIH Publication 08-5846. Available at www.nhlbi.nih.gov/
guidelines/asthma/. Accessed September 24, 2011.

1. Mild-moderate exacerbation (FEV1 of 40% or more)


a. Oxygen to achieve Sao2 of 90% or more
b. An inhaled SABA (MDI with valved holding chamber or nebulizer) up to three doses in the first hour
i. Adult dose: Albuterol MDI 4–8 puffs every 20 minutes for up to 4 hours; then every 1–4
hours as needed or by nebulizer 2.5–5 mg every 20 minutes for three doses; then 2.5–10 mg
every 1–4 hours as needed
ii. Pediatric dose (12 years or younger): Albuterol MDI 4–8 puffs every 20 minutes for three
doses; then every 1–4 hours as needed; use holding chamber (add mask if younger than 4
years old) or by nebulizer 0.15 mg/kg (minimal dose 2.5 mg) every 20 minutes for three
doses; then 0.15–0.3 mg/kg up to 10 mg every 1–4 hours as needed
c. OCS if no response immediately or if patient recently took OCS
2. Severe exacerbation (FEV1 less than 40%)
a. Oxygen to achieve Sao2 of 90% or more
b. High-dose inhaled SABA plus ipratropium by MDI plus valved holding chamber or nebulizer)
every 20 minutes or continuously for 1 hour
c. Oral corticosteroids
i. Adult dose: Prednisone 40–80 mg/day in one or two divided doses until peak expiratory flow
reaches 70% of predicted
ii. Pediatric dose (12 years or younger): 1–2 mg/kg in two divided doses (maximum 60 mg/day)
until peak expiratory flow is 70% of predicted
d. Consider adjunctive therapies (intravenous magnesium or heliox) if still unresponsive

ACCP Updates in Therapeutics® 2012: The Pharmacotherapy Preparatory Review and Recertification Course

1-247
Ambulatory Care

3. Impending or actual respiratory arrest


a. Intubation and mechanical ventilation with oxygen 100%
b. Nebulized SABA plus ipratropium
c. Intravenous corticosteroids
d. Consider adjunctive therapies (intravenous magnesium or heliox) if patient is still unresponsive to
therapy.
e. Admit to intensive care.
4. Treatments that are NOT recommended in emergency care or hospital: Methylxanthines, antibiotics
(unless needed for comorbid conditions), aggressive hydration, chest physical therapy, mucolytics, or
sedation

K. Managing Exacerbations – ED or Hospital – After Repeat Assessment


1. Moderate exacerbation (FEV1 40%–69%)
a. Inhaled SABA every 60 minutes
b. Oral corticosteroid
c. Continue treatment for 1–3 hours if improving
2. Severe exacerbation (FEV1 less than 40%); no improvement after initial treatment
a. Oxygen
b. Nebulized SABA plus ipratropium; hourly or continuous
c. Consider adjunctive therapies.
3. If good response to above treatment and maintained for at least 60 minutes
a. Continue inhaled SABA.
b. Continue OCS course.
c. Consider initiation of ICS (if not already on one).
d. Discharge home.
4. If incomplete response (FEV1 40%–69%) – Admit to hospital ward.
5. If poor response (FEV1 less than 40%) – Admit to intensive care.

L. Monitoring
1. Peak flow monitoring
a. Symptom-based and peak flow–based monitoring has similar benefits; either is appropriate for
most patients. Symptom-based monitoring is more convenient.
b. Consider daily home peak flow monitoring for moderate-severe persistent asthma if patient
has history of severe exacerbations or if patient has poor perception of worsening of asthma
symptoms.
c. Personal best PEFR should be determined if using peak flow–based asthma action plan, not
predicted PEFR.
i. Personal best PEFR is the highest number obtained after daily monitoring for 2 weeks 2
times/day when asthma is under good control.
ii. Predicted PEFR is based on population norms using sex, height, and age.
2. Spirometry (if 5 years or older)
a. At initial assessment
b. After treatment is started and symptoms are stabilized
c. If prolonged or progressive loss of asthma control
d. At least every 1–2 years or more often, depending on response to therapy

M. Vaccines – Adults with asthma (ages 19–64) should receive:


1. The 23-valent pneumococcal polysaccharide vaccine (Pneumovax) once; then a one-time revaccination
with pneumococcal vaccine at age 65 if 5 years or more after the first vaccination
2. Influenza vaccine every fall/winter

ACCP Updates in Therapeutics® 2012: The Pharmacotherapy Preparatory Review and Recertification Course

1-248
Ambulatory Care

Patient Cases
10. R.D. is a 25-year-old man who presents to the ED with shortness of breath at rest. He is having trouble with
conversation. He used 4 puffs of his albuterol MDI at home with no resolution of symptoms. His FEV1 is
checked, and it is 38% of predicted. Which one of the following best classifies the severity of his asthma
exacerbation?
A. Mild.
B. Moderate.
C. Severe.
D. Life threatening.

11. Which one of the following is the best initial therapy for R.D. in the ED, in addition to oxygen?
A. Oxygen alone is sufficient.
B. Inhaled albuterol MDI 8 puffs every 20 minutes for 1 hour.
C. Inhaled albuterol plus ipratropium by nebulizer every 20 minutes for 1 hour plus intravenous
corticosteroids.
D. Inhaled albuterol plus ipratropium by nebulizer every 20 minutes for 1 hour plus OCS.

N. Asthma in Pregnancy
1. Asthma may worsen, improve, or stay the same during pregnancy.
2. Asthma may increase the risk of perinatal mortality, hyperemesis, vaginal hemorrhage, preeclampsia,
complicated labor, neonatal mortality, prematurity, and low birth weight infants, especially if
uncontrolled. Risks are relatively small and are not shown in all studies.
3. Medications:
a. Preferred controller: Budesonide ICS (only category B ICS); however, if well controlled on other
ICS before pregnancy, it may be continued
b. Preferred rescue: Albuterol
c. LABAs are category C; less clinical experience. Use during pregnancy is reasonable if necessary
for asthma control. Salmeterol is preferred LABA because of more experience with salmeterol.
d. LTM modifiers have limited data; most data are with montelukast (category B), and the data for
montelukast are reassuring. Considered alternative therapy
e. Prednisone is category C; potential adverse effects in pregnancy are cleft palate, preeclampsia,
gestational diabetes, low birth weight, prematurity. However, few studies were in asthma, and
women might have been exposed to longer-term prednisone use. Prednisone should be used if
necessary for acute exacerbations in pregnancy.

II. CHRONIC OBSTRUCTIVE PULMONARY DISEASE

Guidelines:
Global Initiative for Chronic Obstructive Lung Disease. Global Strategy for Diagnosis, Management and
Prevention of COPD. Global Initiative for Chronic Obstructive Lung Disease (GOLD) 2011 Revision. Available
at www.goldcopd.org/. Accessed March 1, 2012.

Qaseem A, Wilt TJ, Weinberger SE, et al. Diagnosis and Management of Stable Chronic Obstructive Pulmonary
Disease: A Clinical Practice Guideline Update from the American College of Physicians, American College
of Chest Physicians, American Thoracic Society, and European Respiratory Society (ACP/ACCP/ATS/ERS
Guidelines). Ann Intern Med 2011;155:179–91.

ACCP Updates in Therapeutics® 2012: The Pharmacotherapy Preparatory Review and Recertification Course

1-249
Ambulatory Care

A. Definition – COPD is a syndrome of chronic limitation in expiratory airflow encompassing emphysema or


chronic bronchitis. Airflow obstruction may be accompanied by airway hyperresponsiveness and may be
not be fully reversible.
1. Chronic bronchitis consists of persistent cough plus sputum production for most days out of 3
months in at least 2 consecutive years.
2. Emphysema is abnormal permanent enlargement of the airspaces distal to the terminal
bronchioles, accompanied by destruction of their walls and without obvious fibrosis.

B. Diagnosis and Assessment


1. The diagnosis of COPD is based on a history of exposure to risk factors and the presence of airflow
limitation that is not fully reversible, with or without the presence of symptoms.
a. Symptoms: Dyspnea (described by patients as “increased effort to breathe,” “heaviness,” “air
hunger,” or “gasping”), poor exercise tolerance, chronic cough, sputum production, wheezing
b. GOLD guidelines: Perform spirometry and consider COPD if an individual is older than 40 years
and has any of the following:
i. Dyspnea that is progressive (worsens over time), persistent (present every day), and worse on
exercise.
ii. Chronic cough that is present intermittently or every day; often present through the day;
seldom only nocturnal. May be nonproductive
iii. Chronic sputum production in any pattern and/or
iv. History of exposure to risk factors, especially tobacco smoke (most common risk factor),
occupational dusts and chemicals, and smoke from home cooking and heating fuels
c. ACP/ACCP/ATS/ERS guidelines: The single best predictor of airflow obstruction is the presence
of all three of the following:
i. Smoking history of more than 55 pack-years
ii. Wheezing on auscultation
iii. Patient self-reported wheezing
2. For the diagnosis and assessment of COPD, spirometry is the gold standard.
a. Spirometry showing an FEV1/FVC less than 70% of predicted is the hallmark of COPD.
Bronchodilator reversibility testing is no longer recommended.
b. Measuring arterial blood gas tension should be considered for all patients with FEV1 less than 50%
of predicted or clinical signs suggestive of respiratory failure or right heart failure.
3. Validated symptom scales/questionnaires
a. Modified British Medical Research Council breathlessness scale (mMRC) for assessing severity of
breathlessness (Bestall JC, et al. Thorax 1999;54:5816)
b. COPD Assessment Test (CAT) measures health status impairment in COPD (www.catestonline.org).

C. Factors Determining Severity of COPD


1. Severity of symptoms
2. Severity of airflow limitation (FEV1)
3. Frequency of exacerbations
4. Presence of comorbidities

D. Therapy Goals
1. Relieve symptoms.
2. Reduce the frequency and severity of exacerbations.
3. Improve exercise tolerance.
4. Improve health status.
5. Minimize adverse effects from treatment.

ACCP Updates in Therapeutics® 2012: The Pharmacotherapy Preparatory Review and Recertification Course

1-250
Ambulatory Care

E. Management of Stable COPD


1. Description of levels of evidence/grades of recommendations

GOLD Guidelines
A Randomized clinical trials
Rich body of data
B Randomized clinical trials
Limited body of data
C Nonrandomized trials
Observational studies
D Panel judgment consensus
ACP/ACCP/ATS/ERS Guidelines
Recommendation Strong (S):
grade Benefits clearly outweigh risks and burden, or risks and burden clearly outweigh benefits
Weak (W):
Benefits finely balanced with risks and burden
Quality of High (H)
evidence Moderate (M)
Low (L)

2. Existing medications for COPD have not been shown to modify the long-term decline in lung function,
the hallmark of this disease (Evidence A). Therefore, pharmacotherapy for COPD is used to decrease
symptoms, complications, or both.
3. Smoking cessation is a critical component of COPD management.
4. Bronchodilator medications are central to the symptomatic management of COPD (Evidence A).
a. They are given on an as-needed basis or on a regular basis to prevent or reduce symptoms.
b. The principal bronchodilator treatments are β2-agonists, anticholinergics, or a combination of these
drugs (Evidence A). Theophylline is a bronchodilator given on a regular basis.
c. Inhaled therapy is preferred.
d. The choice between a LABA, anticholinergic, theophylline, and combination therapy depends on
availability and individual response in symptom relief and adverse effects.
e. Regular treatment with a long-acting bronchodilator is more effective and convenient than regular
treatment with SABAs (Evidence A).
f. Combining bronchodilators from different pharmacologic classes may improve efficacy with
the same or fewer adverse effects compared with increasing the dose of a single bronchodilator
(Evidence A).
g. Adding a LABA/ICS to tiotropium appears to provide additional benefits (Welte T, et al. Am J
Respir Crit Care Med 2009;180:741–50).
h. All bronchodilators improve symptoms and exercise capacity.
i. Treatment with a long-acting anticholinergic delays first exacerbation, reduces the overall
number of COPD exacerbations and related hospitalizations, improves health status (Evidence
A), and improves the effectiveness of pulmonary rehabilitation (Evidence B). Initial studies
with tiotropium showed increased cardiovascular risk, but newer strong evidence shows no
increase in risk. Anticholinergics may not significantly improve FEV1.
ii. LABAs improve health status, quality of life, and FEV1 and decrease COPD exacerbation rate
(Evidence A). Salmeterol reduces hospitalization rate (Evidence B).

ACCP Updates in Therapeutics® 2012: The Pharmacotherapy Preparatory Review and Recertification Course

1-251
Ambulatory Care

5. ICSs in stable COPD


a. ICSs improve symptoms, lung function, and quality of life and decrease the frequency of
exacerbations in patients with FEV1 less than 60% of predicted; they do not modify the
progressive decline in FEV1 or decrease mortality (Evidence A).
b. The dose-response with ICS in COPD is unknown (in contrast to asthma treatment). Moderate-
high doses have been used in COPD clinical trials.
c. An ICS combined with a LABA is more effective than the individual components (Evidence A).
An ICS-LABA combination reduces the rate of decline of FEV1 and reduces the exacerbation
rate; the reduction in mortality compared with placebo just fell short of statistical significance
(relative risk reduction, 17.5%; absolute risk reduction, 2.6%; adjusted p=0.052) (TORCH
study: Calverley PMA, et al. N Engl J Med 2007;356:775–89). A subsequent meta-analysis
showed that ICS-LABA might reduce mortality (NNT 36) (Evidence B) (Nannini LJ, et al.
Cochrane Database Syst Rev 2007;4).
d. ICS use is associated with an increased incidence of pneumonia in COPD (Am J Respir Crit Care
Med 2007;176:162–6; Arch Intern Med 2009;169:219–29).
e. Monotherapy with ICS is not recommended.
f. Chronic treatment with OCSs should be avoided because of an unfavorable benefit-risk ratio
(Evidence A).
6. Patient assessment and selection of therapy
a. Previous GOLD guidelines categorized COPD severity and recommended treatment of COPD by
postbronchodilator FEV1 alone. Staging is no longer based on FEV1 alone. New GOLD guidelines
combine symptoms (based on symptom scores), airflow limitation (based on postbronchodilator
FEV1), and frequency of exacerbations to determine patient risk group and recommended
treatment.
b. ACP/ACCP/ATS/ERS guidelines simplify treatment even further on the basis of FEV1 in patients
with COPD with symptoms. They do not provide detailed treatment guidelines.

Table 8. GOLD Guidelines: Assessment of COPD Severity/Risk


Patient Spirometric GOLD Exacerbations
Group Characteristic Classificationa per Yeara Symptom Scoreb
A Low risk 1 Mild (FEV1 ≥ 80% of pred.) or ≤1 mMRC 0–1
Less symptoms 2 Moderate (50% ≤ FEV1 CAT < 10
< 80% of pred.)
B Low risk 1 Mild (FEV1 ≥ 80% of pred.) or ≤1 mMRC ≥ 2
More symptoms 2 Moderate (50% ≤ FEV1 CAT ≥ 10
< 80% of pred.)
C High risk 3 Severe (30% ≤ FEV1 ≥2 mMRC 0–1
Less symptoms < 50% of pred.) or CAT < 10
4 Very severe (FEV1 < 30% of pred.)
D High risk 3 Severe (30% ≤ FEV1 ≥2 mMRC ≥ 2
More symptoms < 50% of pred.) or CAT ≥ 10
4 Very severe (FEV1 < 30% of pred.)
a
To determine the risk of exacerbation, either the spirometric GOLD classification or the number of exacerbations per year can be used. If they
are both used and the patient would fall into two different categories, always assign patient to the category with the highest risk/symptoms.
b
CAT score is preferred, but either can be used.
COPD = chronic obstructive pulmonary disease; FEV1 = forced expiratory volume in 1 second; mMRC = modified British Medical Research
Council breathlessness scale (validated questionnaire); CAT = COPD Assessment Test (validated questionnaire).
Adapted from: Global Initiative for Chronic Obstructive Lung Disease. Global Strategy for Diagnosis, Management and Prevention of COPD.
Global Initiative for Chronic Obstructive Lung Disease (GOLD) 2011 Revision. Available at www.goldcopd.org/. Accessed March 1, 2012.

ACCP Updates in Therapeutics® 2012: The Pharmacotherapy Preparatory Review and Recertification Course

1-252
Ambulatory Care

Table 9. Gold Guidelines: Pharmacotherapy for Stable COPD


Patient
Group First Choice Second Choice Alternativesa
SA anticholinergic PRN or LA anticholinergic or Theophyllineb
A SABA PRN LABA or
SABA + SA anticholinergic
LA anticholinergic or LA anticholinergic + LABA SABA and/or
B LABA SA anticholinergic
Theophyllineb
ICS + LABA LA anticholinergic + LABA PDE-4 inhibitor
C or SABA and/or
LA anticholinergic SA anticholinergic
Theophyllineb
ICS + LABA ICS + LA anticholinergic or SABA and/or
or ICS + LABA and LA anticholinergic or SA anticholinergic
D LA anticholinergic ICS + LABA and PDE-4 inhibitor or Theophyllineb
LA anticholinergic + LABA or
LA anticholinergic + PDE-4 inhibitor
a
Alternative medications can be used alone or in combination with first and second choice columns
b
Theophylline is not recommended unless other long-term bronchodilators are unavailable or unaffordable
COPD = chronic obstructive pulmonary disease; ICS = inhaled corticosteroid; LA = long-acting; LABA = long-acting β2-agonist; SABA =
short-acting β2-agonist.
Adapted from: Global Initiative for Chronic Obstructive Lung Disease. Global Strategy for Diagnosis, Management and Prevention of COPD.
Global Initiative for Chronic Obstructive Lung Disease (GOLD) 2011 Revision. Available at www.goldcopd.org/. Accessed March 1, 2012.

Table 10. ACP/ACCP/ATS/ERS Guidelines: Treatment Recommendations for Stable COPD

• For patients with respiratory symptoms and FEV1 between 60% and 80% of predicted, treatment with long-
acting inhaled bronchodilators is suggested.
(Grade: W, Evidence: L)

• For patients with respiratory symptoms and FEV1 < 60% of predicted, treatment with long-acting inhaled
bronchodilators is recommended.
(Grade: S, Evidence: M)

• Monotherapy using either long-acting inhaled anticholinergics or LABAs is recommended for symptomatic
patients with FEV1 < 60% of predicted. The choice of specific monotherapy should be based on patient
preference, cost, and adverse effect profile.
(Grade: S, Evidence: M)

• Combination inhaled therapies (long-acting inhaled anticholinergics, LABAs, or ICS) may be used for
symptomatic patients FEV1 < 60% of predicted.
(Grade: W, Evidence: M)
Qaseem A, et al. ACP/ACCP/ATS/ERS COPD guidelines. Ann Intern Med 2011;155:179–91.

ACCP Updates in Therapeutics® 2012: The Pharmacotherapy Preparatory Review and Recertification Course

1-253
Ambulatory Care

Patient Cases
12. S.H. is a 50-year-old male smoker with a recent diagnosis of COPD. Spirometry showed FEV1/FVC 60%;
pre-bronchodilator FEV1 70% of predicted; and postbronchodilator FEV1 72% of predicted. His symptoms
are very bothersome, and he reports walking slower than others because of shortness of breath and having
to stop to catch his breath every so often when walking on level ground (mMRC grade 2). He had one exac-
erbation in the past year. Which of the following is the most appropriate patient group for S.H., according
to the GOLD guidelines?
A. Patient group A.
B. Patient group B.
C. Patient group C.
D. Patient group D.

13. Which one of the following pharmacotherapy options is most appropriate for S.H. to be initiated on?
A. Albuterol MDI 2 puffs every 4–6 hours as needed.
B. Albuterol MDI 2 puffs every 4–6 hours as needed plus formoterol inhale 1 capsule 2 times/day.
C. Albuterol MDI 2 puffs every 4–6 hours as needed plus salmeterol/fluticasone 50/500 1 puff 2 times/day.
D. Albuterol MDI 2 puffs every 4–6 hours as needed plus salmeterol/fluticasone 50/500 1 puff 2 times/
day plus Daliresp 500 mcg orally once daily.

14. K.R. is a 60-year-old man with COPD who smokes ½ pack/day (cut down from 2 packs/day). He has had a
gradual worsening in shortness of breath during the past 1–2 years. Spirometry shows FEV1/FVC of 55%
and FEV1 of 63%. His CAT score is 10. He has not had a COPD exacerbation or received systemic cortico-
steroids in the past 2 years. His current COPD medications are tiotropium (Spiriva) once daily and albuterol
HFA as needed. According to the GOLD guidelines, which one of the following is the most appropriate
course of action?
A. Add salmeterol 1 puff 2 times/day.
B. Add long-term azithromycin 250 mg once daily.
C. Add fluticasone 110 mcg 2 puffs 2 times/day.
D. Discontinue tiotropium and start Advair 250/50.

7. Other pharmacologic treatments


a. Phosphodiesterase-4 inhibitor: Roflumilast (Daliresp)
i. Approved by the FDA in March 2011
ii. Indication: As a daily treatment to reduce the risk of COPD exacerbations in patients with
severe COPD (FEV1 less than 50% of predicted) associated with chronic bronchitis and a
history of frequent exacerbations. In these patients, studies show a reduction in exacerbations
and a reduction in the composite end point of moderate exacerbations treated with oral
or systemic corticosteroids or severe exacerbations requiring hospitalization or causing
death (Evidence B). These effects are also seen when roflumilast is added to long-acting
bronchodilators (Evidence B). No comparison has been done with ICSs.
iii. Mechanism: Reduces inflammation through inhibition of the breakdown of intracellular cyclic
adenosine monophosphate; no direct bronchodilator activity
iv. Dose: 500 mcg orally once daily
v. Contraindications: Moderate to severe liver impairment; use in nursing mothers
vi. Precautions: Weight loss (monitor), psychiatric events including suicidality (monitor; weigh
risks versus benefits in patients with preexisting psychiatric illness). 20% of patients studied

ACCP Updates in Therapeutics® 2012: The Pharmacotherapy Preparatory Review and Recertification Course

1-254
Ambulatory Care

had weight loss of 5%–10% of body weight compared with 7% with placebo; average weight
loss was 2 kg.
vii. Adverse reactions: Diarrhea, weight loss/decreased appetite, nausea, headache, back pain,
influenza, insomnia, and dizziness
viii. Drug interactions: Use with strong cytochrome P450 (CYP) enzyme inducers is not
recommended (e.g., rifampin, phenobarbital, carbamazepine, phenytoin); use with CYP3A4
inhibitors or dual inhibitors of CYP 3A4 and 1A2 (e.g., erythromycin, ketoconazole,
fluvoxamine, enoxacin, cimetidine) increases roflumilast exposure and adverse effects (risk-
benefit must be weighed).
b. Smoking cessation (essential for all patient groups A–D)
c. Influenza vaccine annually (essential for all patient groups A–D)
d. Pneumococcal vaccine (essential for all patient groups A–D)
e. α1-Antitrypsin augmentation therapy (Evidence C)
i. For young patients with severe hereditary α1-antitrypsin deficiency and established
emphysema, but an expensive treatment
ii. Patients with α1-antitrypsin deficiency usually are white, usually develop COPD at a young
age (younger than 45 years), and have a strong family history of it. It may be worthwhile
screening such patients.
f. β-Blockers
i. New observational data suggest that long-term treatment with β-blockers reduces risk of
exacerbations and improves survival, even in patients without overt cardiovascular disease
(Rutten FH, et al. Arch Intern Med 2010;170:880–7).
ii. More than half of the patients in the study had cardiovascular risk factors or coronary artery
disease. Mostly, cardioselective β-blockers were used.
iii. It is too early to recommend β-blockers for the treatment of COPD, but β-blockers should
not be withheld in patients with COPD who also have heart disease, chronic heart failure
(CHF), or other cardiovascular conditions in which β-blockers are beneficial (Salpeter SR,
et al. Cardioselective beta blockers for chronic obstructive pulmonary disease. Cochrane
Database Syst Rev 2002;2:CD003566. Review. Update in: Cochrane Database Syst Rev 2005.
Reviewed 2008;4:CD003566)
iv. Mechanism for benefit in COPD is unknown, but β-blockers can up-regulate β2-receptors in
the lungs, which may improve the effectiveness of inhaled β-agonists.
g. Antibiotics are recommended only for treating infectious exacerbations of COPD.
8. Nonpharmacologic therapy
a. Home oxygen therapy
i. Recommended in patients who have Pao2 of 55 mm Hg or less (or 55–60 mm Hg if
pulmonary hypertension, peripheral edema, or polycythemia; Evidence D) or Sao2 of 88% or
less, with or without hypercapnia, confirmed twice during a 3-week period (Evidence B)
ii. Long-term (more than 15 hours/day) use in patients with chronic respiratory failure improves
survival.
b. Pulmonary rehabilitation (essential for patient groups B–D; Evidence A)
i. Includes exercise training, nutrition counseling, and education
ii. Recommended for stage II–IV COPD. Patients should be referred when they have moderate
(stage II) COPD, not wait until it is more severe.
iii. Has been shown to improve many outcomes in COPD, including quality of life and survival
9. New data in COPD
a. Tiotropium versus salmeterol for COPD
i. POET-COPD study (N Engl J Med 2011;364:1093–103)

ACCP Updates in Therapeutics® 2012: The Pharmacotherapy Preparatory Review and Recertification Course

1-255
Ambulatory Care

ii. 736 patients with moderate-severe COPD and one or more exacerbations in past year; 1-year
randomized, double-blind, parallel-group trial
iii. Tiotropium versus salmeterol
iv. Primary end point: Time to first exacerbation
(a) Moderate exacerbation: treated with OCS, antibiotics, or both
(b) Severe exacerbation: hospitalized
v. Results: Tiotropium (vs. salmeterol) significantly:
(a) Increased time to first exacerbation (187 days vs. 145 days [42-day difference]); both
moderate and severe exacerbations were significant
(b) Reduced annual number of exacerbations; rate of both moderate and severe exacerbations
was significant
(c) Benefit was consistent in all major subgroups and independent of concomitant ICS.
(d) Benefit evident in 1 month and maintained for 1 year
(e) Significantly fewer patients taking tiotropium withdrew early.
vi. Conclusion: Tiotropium may be more effective than LABAs as initial long-acting
bronchodilator therapy.
b. Chronic azithromycin for prevention of COPD exacerbations
i. Albert RK, et al. N Engl J Med 2011;365:689–98
ii. 1577 subjects at increased risk of exacerbations (stage II [moderate] or worse COPD either
on continuous O2 or received systemic corticosteroids in past year, and history of a COPD
exacerbation requiring ED visit or hospitalization; no history of hearing impairment)
iii. Subjects randomized to daily azithromycin 250 mg or placebo for 1 year
iv. Results:
(a) Median time to exacerbation: 266 days (azithromycin group) versus 174 days (placebo)
(p<0.001)
(b) Rate of acute exacerbation: 1.48 vs. 1.83 for azithromycin versus placebo (p=0.01)
(c) Number needed to treat (NNT) to prevent one acute exacerbation of COPD: 2.86
(d) Quality of life improved more with azithromycin versus placebo (based on St. George’s
Respiratory Questionnaire; p=0.03)
(e) However, hearing decrements (by audiometry) were more common with azithromycin
versus placebo (25% vs. 20%, p=0.04) (NNH [number needed to harm] = 20), and in the
azithromycin group, there was an increased incidence of colonization with macrolide-
resistant organisms (81% vs. 41%, p<0.001).
v. Conclusion: Daily azithromycin lengthens time to first exacerbation, decreases rate of
exacerbations, and improves quality of life in patients with COPD at increased risk of
exacerbations, with the expense of risk of hearing decrements and increasing macrolide-
resistant organism colonization. The most recent GOLD guidelines still do not recommend
treatment with antibiotics, except for when indicated during acute exacerbations.

F. Management of Acute Exacerbations of Chronic COPD


1. A COPD exacerbation is an acute worsening of a patient’s baseline respiratory symptoms (dyspnea and/
or cough and/or an increase in quantity or purulence of sputum) that is worse than normal day-to-day
variation and results in a change in medication. Diagnosis is based purely on clinical presentation.
2. Common precipitation factors include infection of tracheobronchial tree and viral upper respiratory
tract infections (most common) and air pollution, but the cause of one-third of exacerbations cannot be
determined.
3. Spirometry is not accurate during an exacerbation and is not recommended.
4. Pulse oximetry can be used to determine the need for supplemental oxygen, which should be given in

ACCP Updates in Therapeutics® 2012: The Pharmacotherapy Preparatory Review and Recertification Course

1-256
Ambulatory Care

severe exacerbations. In exacerbations requiring hospitalization, an arterial blood gas measurement


should be done.
5. Inhaled bronchodilators (inhaled SABAs with or without short-acting anticholinergics) are the
preferred treatment of COPD exacerbations (Evidence C).
a. Usual doses of albuterol are 2.5 mg by a nebulizer every 1–4 hours as needed or 4–8 puffs by MDI
with holding chamber every 1–4 hours as needed.
b. Short-acting anticholinergics (ipratropium) are generally added for acute exacerbation.
6. Systemic corticosteroids are effective, and they shorten recovery time, improve FEV1 and improve
hypoxemia (Evidence A). They may also lower the risk of treatment failure, early relapse rate,
and length of hospital stay. Systemic corticosteroids should be used in most exacerbations; GOLD
guidelines no longer provide criteria for use. Dose of OCSs for outpatient treatment: 30–40 mg of oral
prednisolone (or equivalent) once daily for 10–14 days (Evidence D)
7. Antibiotic treatment should be initiated for exacerbations if criteria below are met. Most common
pathogens in COPD exacerbations: Streptococcus pneumoniae, Haemophilus influenzae, and
Moraxella catarrhalis. In patients with Gold 3 and 4 severity, Pseudomonas aeruginosa infection
becomes an important pathogen.
a. The three cardinal symptoms in COPD exacerbations are increased dyspnea, increased sputum
volume, and increased sputum purulence.
i. Antibiotics should be given if all three cardinal symptoms are present (Evidence B).
ii. Antibiotics should be given if two of the three cardinal symptoms are present AND if
increased sputum purulence is one of the symptoms (Evidence C).
iii. Antibiotics should be given to patients with a severe exacerbation requiring mechanical
ventilation (Evidence B).
b. Recommended duration of antibiotic treatment is usually 5–10 days (Evidence D).
c. Recommended antibiotics:
i. Optimal antibiotic therapy has not been determined. Usual initial antibiotics for
uncomplicated COPD: Azithromycin, clarithromycin, doxycycline, or trimethoprim/
sulfamethoxazole. In complicated COPD with risk factors: Amoxicillin/clavulanate,
levofloxacin, moxifloxacin. If at risk of Pseudomonas infection: High-dose levofloxacin (750
mg) or ciprofloxacin
ii. If exacerbation does not respond to initial antibiotic, then sputum culture and sensitivity
should be performed.
G. Vaccinations: All patients with COPD should receive the influenza vaccine yearly and the polysaccharide
pneumococcal vaccine once before age 65; then a one-time revaccination 5 years or more after the first
vaccination

Patient Case
15. M.J. is a 56-year-old man with COPD in GOLD patient group A who presents to the clinic with a several-
day history of worsening shortness of breath, coughing that is keeping him up at night, and production of
“cloudy” sputum – much more sputum than usual. Pulse oximetry is 95% on room air. He has a nebulizer
at home. In addition to regular use of albuterol plus ipratropium by nebulizer every 1–4 hours, which one of
the following is the best course of action?
A. No additional therapy is needed.
B. Add oral prednisone 40 mg once daily for 10 days.
C. Add trimethoprim/sulfamethoxazole double strength 1 tablet twice daily for 7 days.
D. Add oral prednisone 40 mg once daily for 10 days and trimethoprim/sulfamethoxazole double
strength 1 tablet twice daily for 7 days.

ACCP Updates in Therapeutics® 2012: The Pharmacotherapy Preparatory Review and Recertification Course

1-257
Ambulatory Care

III. ANTICOAGULATION

Guidelines:
Guyatt GH, Akl EA, Crowther M, Gutterman DD, Schuunemann HF. Antithrombotic Therapy and Prevention of
Thrombosis, 9th ed. American College of Chest Physicians Evidence-Based Clinical Practice Guidelines. Chest
2012;141:2(Suppl):1S–70S.

A. Warfarin Products
1. Brands: Coumadin, Jantoven, and many generics (Barr, Geneva, Taro)
2. Strengths: 1-, 2-, 2.5-, 3-, 4-, 5-, 6-, 7.5-, and 10-mg tablets. All are scored.
3. Tablet colors for each strength are the same between all manufacturers.

B. Mechanism of Action of Warfarin


1. Inhibits vitamin K–dependent clotting factors II, VII, IX, and X
2. Inhibits anticoagulant proteins C and S
3. Racemic mixture of R- and S-isomers
a. S-isomer more potent vitamin K antagonist
b. S-isomer metabolized primarily by CYP2C9
c. R-isomer metabolized primarily by CYP3A4

C. Indications

Table 11. Recommendations for Long-term Warfarin Vitamin K Antagonist (VKA) Therapya
Thromboembolic Disorder INR Duration Evidence Comments
Venous Thromboembolism (VTE): DVT or PE
DVT/PE with reversible or 2.5 (2–3) 3 months 1A Time-limited risk factors:
time-limited risk factor Surgery, immobilization,
estrogen, pregnancy
For all acute DVTs, initiate
LMWH or fondaparinux
(preferred) or UFH; start
VKA on day 1 if possible,
and continue parenteral
anticoagulation for at least
5 days and until INR ≥ 2
for at least 24 hours (1B).
If LMWH used, once daily
suggested over twice daily
(2C) if approved once-daily
regimen has the same daily
dose as the twice-daily
regimen (e.g., dalteparin)

ACCP Updates in Therapeutics® 2012: The Pharmacotherapy Preparatory Review and Recertification Course

1-258
Ambulatory Care

Table 11. Recommendations for Long-term Warfarin Vitamin K Antagonist (VKA) Therapya (continued)
Thromboembolic Disorder INR Duration Evidence Comments
First unprovoked PE or 2.5 (2–3) At least 3 1A Same as above for initiating
proximal DVT months; then VKA together with
evaluate for parenteral anticoagulation.
risk-benefit Unprovoked isolated distal
ratio of DVT: 3 months is sufficient.
extended Treat recurrence (second-
therapy. episode unprovoked VTE)
Proximal DVT with extended therapy
or PE: if low (1B) or moderate
If low-moderate (2B) bleeding risk.b
bleeding riskb: Treat recurrence of unprovoked
extended VTE with 3 months VKA
VKA (2B). over extended therapy if
If high bleeding high bleeding riskb (2B).
riskb: 3 months DVT with cancer: extended
VKA (1B) LMWH (preferred) or
VKA if bleeding risk is not
high (1B) or highb (2B)
CAD/Acute MI
Anterior MI and LV thrombus, or 2–3 3 months of 1B VKA plus ASA (75–100
at high risk of LV thrombus (EF VKA mg/day) for first 3 months
< 40%, anteroapical wall motion (1B); then switch to dual
abnormality) without stenting antiplatelet therapy for up
to 12 months, followed by
single antiplatelet therapy
Anterior MI and LV thrombus, 2–3 3 months of 2C Triple therapy (VKA, ASA,
or at high risk of LV thrombus VKA clopidogrel) for 1 month; then
(EF < 40%, anteroapical wall VKA plus single antiplatelet
motion abnormality) with BMS agent for 2 months; then D/C
VKA and switch to dual
antiplatelet therapy for up
to 12 months, followed by
single antiplatelet therapy
Anterior MI and LV thrombus, 2C Triple therapy (VKA, ASA,
or at high risk of LV thrombus clopidogrel) for 3–6 months;
(EF < 40%, anteroapical wall then D/C VKA and continue
motion abnormality) with DES dual antiplatelet therapy for
up to 12 months, followed by
single antiplatelet therapy
Atrial Fibrillation (including paroxysmal/intermittent)/Atrial Flutter
AF and low stroke risk N/A N/A 2B No therapy. If patients choose
(CHADS2 score = 0)
c
antithrombotic therapy, use
ASA (75–325 mg/day)

ACCP Updates in Therapeutics® 2012: The Pharmacotherapy Preparatory Review and Recertification Course

1-259
Ambulatory Care

Table 11. Recommendations for Long-term Warfarin Vitamin K Antagonist (VKA) Therapya (continued)
Thromboembolic Disorder INR Duration Evidence Comments
AF and intermediate stroke Long term 1B Dabigatran 150 mg BID
risk (CHADS2 scorec = 1) suggested over VKA (2B).
2.5 (2–3) Long term If stable CHD (no ACS in past
year) or mitral stenosis, use
AF and high stroke risk Long term 1A VKA (no addition of ASA).
(CHADS2 scorec ≥ 2) 2.5 (2–3) Long term In patients who choose not
to take oral anticoagulants
(for reasons other than
bleeding), use combination
ASA/clopidogrel
AF with low-intermediate 2.5 (2–3) Long term 2C Dual antiplatelet treatment
stroke risk (CHADS2 scorec (e.g., ASA and clopidogrel) for
0–1) and stent placement 12 months (BMS and DES),
followed by VKA long term
AF with high stroke risk (CHADS2 2.5 (2–3) Long term 2C Triple therapy with VKA,
scorec ≥ 2) and stent placement ASA, and clopidogrel for
1 month (BMS) or 3–6
months (DES), followed
by VKA plus single
antiplatelet agent long term
AF with low stroke risk 2.5 (2–3) Long term 2C Dual antiplatelet (e.g., ASA
(CHADS2 scorec = 0) and after plus clopidogrel) and no
acute ACS without stent first 12 VKA for first 12 months; then
months change to VKA alone (no
addition of antiplatelet agent)
AF with intermediate-high stroke 2.5 (2–3) Long term 2C VKA plus single antiplatelet
risk (CHADS2 scorec ≥ 1) and agent for 12 months,
acute ACS without stent followed by VKA alone
(no addition of ASA)
Conversion of Atrial Fibrillation
Elective direct current 2.5 (2–3) ≥ 7 weeks 1C Other (equal) options:
cardioversion of AF ≥ 48 dabigatran or LMWH
hours or unknown duration (treatment dose).
At least 3 weeks before and
at least 4 weeks after sinus
rhythm maintained.
If AF is < 48 hours in
duration, cardioversion can be
performed without prolonged
anticoagulation; LMWH
(full treatment dose) or UFH
should be used at presentation
Valvular Heart Disease
Rheumatic mitral valve disease 2.5 (2–3) Long term 1A
with history of systemic
embolism, LA thrombus, or AF

ACCP Updates in Therapeutics® 2012: The Pharmacotherapy Preparatory Review and Recertification Course

1-260
Ambulatory Care

Table 11. Recommendations for Long-term Warfarin Vitamin K Antagonist (VKA) Therapya (continued)
Thromboembolic Disorder INR Duration Evidence Comments
Rheumatic mitral valve 2.5 (2–3) Long term 2C
disease in sinus rhythm if
LA diameter is > 55 cm
Mechanical and Bioprosthetic Heart Valves
Bioprosthetic heart valve, mitral 2.5 (2–3) 3 months; then 2C Switch to ASA after 3
ASA 50–100 months if sinus rhythm
mg/day For aortic bioprosthetic valve
and normal sinus rhythm:
ASA 50–100 mg/day (1B)
For any bioprosthetic valve
with risk factors (low EF,
AF, hypercoagulable),
use long-term VKA
Mechanical prosthetic heart 2.5 (2–3) Long term 1B Add ASA (50–100 mg/day) if
valve: aortic low risk of bleeding (1B).
If normal LA size and in
sinus rhythm. Valves: St.
Jude bileaflet, Carbomedics
bileaflet, Medtronic
tilting disk. 2.5–3.5 if AF
or LA enlargement
Mechanical prosthetic 3 (2.5– Long term 2C Add ASA (50–100 mg/day) if
heart valve: mitral 3.5) low risk of bleeding (1B).
Tilting disk or bileaflet
Antiphospholipid Antibody Syndrome
Thromboembolism with lupus 2.5 (2–3) 1A
inhibitor and no additional
risk factors, with no lack
of response to therapy
Previous arterial or venous 2.5 (2–3) 2B
thromboembolic events
a
Evidence categories: 1 = strong, experts are certain; 2 = weaker, experts are less certain; A = consistent results from RCTs; B = inconsistent
results from RCTs; C+ = observational studies with very strong effects or generalizations from RCTs; C = observational studies.
b
Assessment of bleeding risk is based on risk factors (RFs). Low risk: 0 RF, moderate risk: 1 RF, high risk ≥ 2 RF. Risk factors: age > 65, age >
75, previous bleeding, cancer, metastatic cancer, renal failure, liver failure, thrombocytopenia, previous stroke, diabetes, antiplatelet therapy, poor
anticoagulant control, comorbidity and reduced functional capacity, recent surgery, frequent falls, alcohol abuse (bleeding risk from each risk factor
depends on the severity of the RF, the time interval, and whether RF was corrected).
c
CHADS2 score is calculated by assigning points for the following risk factors: CHF (any history), HTN, age ≥ 75, diabetes (1 point each); stroke,
TIA, systemic embolism history (2 points).
AF = atrial fibrillation; ASA = acetylsalicylic acid; BMS = bare metal stent; CAD = coronary artery disease; CHF = chronic heart failure; CV
= cardiovascular; DES = drug-eluting stent; DVT = deep venous thrombosis; EF = ejection fraction; HTN = hypertension; INR = international
normalized ratio; LA = left atrial; LV = left ventricular; LMWH = low-molecular-weight heparin; MI = myocardial infarction; MVP = mitral valve
prolapse; PE = pulmonary embolism; QD = every day; RCT = randomized controlled trial; TIA = transient ischemic attack; VKA = vitamin K
antagonist (warfarin).
Guyatt GH, Akl EA, Crowther M, Gutterman DD, Schuunemann HF. Antithrombotic Therapy and Prevention of Thrombosis, 9th ed. American
College of Chest Physicians Evidence-Based Clinical Practice Guidelines. Chest 2012;141:2(Suppl):1S–70S.

ACCP Updates in Therapeutics® 2012: The Pharmacotherapy Preparatory Review and Recertification Course

1-261
Ambulatory Care

D. Dosing and Administration of Warfarin


1. Overlap with heparin or therapeutic-dose LMWH (see Management of Oral Anticoagulation During
Invasive Procedures section below) for at least 5 days for acute venous thromboembolism (VTE), to
fully inhibit factor II, until INR is therapeutic for at least 24 hours.
2. Initial dose
a. Begin warfarin on day 1 (preferred) or day 2 of heparin or LMWH.
b. CHEST guidelines suggest initiating warfarin at 10 mg/day for the first 2 days, followed by INR-
based dosing in patients healthy enough to be treated as outpatients (2C). Otherwise, 5 mg/day is
generally sufficient.
c. Specific patients may need a lower starting dose (5 mg/day or in some cases 2–3 mg/day).
i. Elderly
ii. Very low body weight
iii. Concurrent interacting drug (enzyme inhibitor)
iv. Malnourished or nothing by mouth for more than 3 days
v. Liver disease
vi. Congestive heart failure
vii. High risk of bleeding
viii. Prolonged fever
ix. Hyperthyroid state
x. Low albumin
3. Adjusting dose
a. Watch for trends; remember that the INR seen today is the result of the doses given during the past
4–5 days. Takes 5–7 days to reach full effect, given the half-life of factor II
b. Half-lives of vitamin K–dependent clotting factors
VII = 6 hours
IX = 24 hours
X = 36 hours
II = 72 hours
4. If INR is out of therapeutic range, increase or decrease cumulative weekly warfarin dose by 5%–20%
depending on INR; if INR is high, may hold one or two doses and resume at a lower dose
a. If INR previously stable/therapeutic and single out-of-range INR is 0.5 or less above or below
therapeutic range, current dose can be continued, and recheck INR within 1–2 weeks (2C)
b. Generally, do not need to adjust if INR is within 0.1 of goal (but monitor more closely).
c. CHEST guidelines recommend AGAINST bridging with LMWH/UFH for a single subtherapeutic INR
in a patient normally stable (2C).
5. Variable INR
a. Some patients on long-term warfarin have a variable INR response that is not from usual known
causes.
b. CHEST guidelines recommend AGAINST the routine use of vitamin K supplementation (2C)
6. Genetic testing
a. A variety of genetic mutations for VKORC1 and CYP2C9 exist.
b. A VKORC1 mutation alters or changes sensitivity or resistance to warfarin.
c. CYP2C9 polymorphisms alter the rate of metabolism and the half-life of warfarin.
d. Mutations of either of these result in an altered initial dose and maintenance dose.
e. The CHEST guidelines recommend against the routine use of pharmacogenetic testing to guide the
dosing of warfarin (1B).

ACCP Updates in Therapeutics® 2012: The Pharmacotherapy Preparatory Review and Recertification Course

1-262
Ambulatory Care

E. Adverse Effects of Warfarin


1. Bleeding
a. Incidence
i. All: 2.4%–29%
ii. Fatal or life threatening: 2%–8%
b. Epistaxis, hematuria, gastrointestinal hemorrhage, bleeding gums
c. Easy bruising often occurs with therapeutic INR.
2. Skin necrosis (rare)
3. Purple toe syndrome (rare)
4. Teratogenicity (category X)
a. First-trimester exposure: Nasal hypoplasia, small nares, chondrodysplasia
b. Second- and third-trimester exposure: Mental retardation, blindness, spasticity, seizures
c. If pregnant, subcutaneous heparin or LMWH is safe to use.
d. Breastfeeding: Can use warfarin because not excreted in breast milk

F. Patient Education with Warfarin


1. Indication and expected therapy duration
2. Bleeding symptoms/signs
3. Management of bleeding complications – When to call health care provider
4. Need for and importance of frequent blood tests and INR goals
5. Drug interactions (especially OTCs [over-the-counter medications] and herbals)
6. Dosing/importance of adherence/pillbox
7. How to take; what to do if a dose missed
8. Dietary instructions; keep intake of vitamin K–containing foods consistent
9. Teratogenicity; need for reliable contraception if of childbearing age
10. Wear medic alert bracelet/necklace or carry identification card.

G. Drug Interactions with Warfarin (lists not all-inclusive; many drug interactions exist)
** Drug interactions are very common with warfarin. The magnitude of interactions varies considerably; for
some, concomitant use is contraindicated; others require a warfarin dose adjustment; for others, more frequent
monitoring of the INR is sufficient.**
1. Reduced warfarin absorption (e.g., cholestyramine, sucralfate)
2. Enzyme induction (decreases INR and warfarin effects)
a. CYP3A4 (e.g., rifampin, carbamazepine, phenobarbital, St. John’s wort)
b. Other (e.g., griseofulvin, nafcillin, dicloxacillin, phenytoin [inhibition; then induction])
c. Delay in onset and offset
3. Enzyme inhibition (­increases INR and warfarin effects)
a. S-warfarin* (CYP2C9) (e.g., metronidazole, trimethoprim/sulfamethoxazole, fluconazole,
isoniazid, fluoxetine, sertraline, amiodarone, clopidogrel, lovastatin)
b. R-warfarin (CYP3A3/4/5) (e.g., omeprazole, clarithromycin, erythromycin, “azole” antifungals,
nefazodone, fluoxetine, amiodarone, cyclosporine, sertraline, grapefruit juice, ciprofloxacin,
norfloxacin, protease inhibitors, diltiazem, verapamil, isoniazid, metronidazole)
4. Antiplatelet effects (e.g., gingko, garlic, aspirin, nonsteroidal anti-inflammatory drugs (NSAIDs),
selective serotonin reuptake inhibitors); NSAIDs and aspirin also increase the risk of ulcers, providing
a site from which to bleed. CHEST guidelines recommend to avoid concomitant antiplatelet drugs
unless benefit outweighs harm (2C).
5. Reduced clearance of warfarin (e.g., propafenone)
6. Increased degradation of clotting factors (e.g., levothyroxine)

ACCP Updates in Therapeutics® 2012: The Pharmacotherapy Preparatory Review and Recertification Course

1-263
Ambulatory Care

7. CHEST guidelines recommend to avoid concomitant NSAIDs (including COX-2 inhibitors) and
antibiotics (see Table 8 in main article and add list) (2C). Antibiotics with evidence of increased risk
of bleeding: trimethoprim/sulfamethoxazole, quinolones, metronidazole, cephalosporins, doxycycline,
amoxicillin, and amoxicillin/clavulanate

Table 12. Management of Increased INR


INR Patient Situation Action
No evidence of Recommend against routine vitamin K
4.5–10 and bleeding Omit one or two doses of warfarin, and monitor INR more often. Restart
warfarin at a lower dose when the INR falls into the therapeutic range
No evidence of Omit the next several warfarin doses and give oral vitamin K1 (5–10
> 10 and bleeding mg). Monitor INR closely and repeat vitamin K1, if necessary.
Resume warfarin at a lower dose when INR is in the desired range
VKA-associated Hold warfarin, give vitamin K1 5–10 mg by slow IV injection
major bleeding plus prothrombin complex concentrate (recommended over
FFP). If needed, repeat vitamin K1 injection every 12 hours
FFP = fresh frozen plasma; INR = international normalized ratio; IV = intravenous.
Adapted from: Guyatt GH, Akl EA, Crowther M, Gutterman DD, Schuunemann HF. Antithrombotic Therapy and Prevention of Thrombosis,
9th ed. American College of Chest Physicians Evidence-Based Clinical Practice Guidelines. Chest 2012;141:2(Suppl):1S–70S.

Patient Cases
16. J.J. is a 30-year-old woman who is receiving warfarin for a proximal DVT. She was taking combination oral
contraceptives at the time her DVT was diagnosed; they have since been discontinued. Which one of the
following is the most appropriate duration of warfarin?
A. 3 months.
B. 6 months.
C. 1 year.
D. Indefinite.

17. M.H. is a 63-year-old woman with a history of mechanical mitral valve replacement, hypertension, and
dyslipidemia. Her medications include warfarin 8 mg/day, aspirin 81 mg/day, lisinopril 20 mg/day, and
atorvastatin 10 mg/day. Which one of the following best represents M.H.’s goal INR?
A. 1.5–2.5.
B. 1.8–2.6.
C. 2.0–3.0.
D. 2.5–3.5.

18. What if M.H. had an aortic mechanical valve (not a mitral valve replacement)? Which one of the following
would best represent M.H.’s goal INR?
A. 1.5–2.5.
B. 1.8–2.6.
C. 2–3.
D. 2.5–3.5.

ACCP Updates in Therapeutics® 2012: The Pharmacotherapy Preparatory Review and Recertification Course

1-264
Ambulatory Care

Questions 19 and 20 pertain to the following case.


B.D. is a 79-year-old man taking warfarin 5 mg/day for atrial fibrillation. His other conditions are depression and
gastroesophageal reflux disease. His medications include fluoxetine 20 mg (started 1 month ago) and omeprazole
40 mg/day (started 6 months ago). He has warfarin 5-, 2-, and 1-mg tablets at home. He has not had his INR
checked for 6 weeks; it was in range at that time. He denies any signs or symptoms of bleeding. You check his
INR, and it is 8.

19. Which one of the following is the best action to deal with this patient’s high INR?
A. Hold warfarin for 1 day and then restart at a lower dose (no need to recheck INR).
B. Hold warfarin for 2 days and then restart at a lower dose (no need to recheck INR).
C. Hold warfarin for 2 days, recheck INR, and restart warfarin at a lower dose when INR approaches 3.
D. Hold warfarin, give oral vitamin K 2.5 mg for 1 day, and restart warfarin at a lower dose when INR
approaches 3.

20. When warfarin is reinitiated in B.D., which of the following is the best dose to start?
A. 5 mg 2 days/week and 2.5 mg 5 days/week.
B. 2.5 mg/day.
C. 4 mg/day.
D. 4.5 mg 2 days/week and 5 mg 5 days/week.

21. A 77-year-old man has atrial fibrillation, hypertension, diabetes, and a history of transient ischemic attack
(TIA) 3 years ago. He will be undergoing major abdominal surgery in 1 week and will need to hold his war-
farin. Which one of the following is the most appropriate LMWH bridge therapy for him?
A. No bridge LMWH is needed; just hold warfarin.
B. Enoxaparin 30 mg 2 times/day.
C. Enoxaparin 1 mg/kg 2 times/day.
D. Enoxaparin 30 mg 2 times/day or 1 mg/kg 2 times/day is an option.

ACCP Updates in Therapeutics® 2012: The Pharmacotherapy Preparatory Review and Recertification Course

1-265
Ambulatory Care

H. Food Interactions with Warfarin

Table 13. Foods with Medium-High Vitamin K Content


Food Rank of Vitamin K
Meat Liver (chicken, beef) High
Dairy Cheese Medium
Grains Oats Medium
Beverages Green tea Higha
Vegetables Asparagus Medium
Avocado Medium
Broccoli High
Brussel sprouts High
Cabbage High
Cauliflower High
Chick peas High
Collard greens High
Green peas Medium
Kale High
Lettuce (iceberg) Medium
Lettuce (Bibb, red leaf, green leaf) High
Mustard greens (raw) High
Pickles Medium
Spinach High
Swiss chard (raw) High
Turnip greens (raw) High
a
Some controversy exists regarding whether brewing green tea decreases its vitamin K content.
U.S. Department of Agriculture. Provisional Table on the Vitamin K Content of Foods. Available at
www.nal.usda.gov/fnic/foodcomp/Data/Other/pt104.pdf. Accessed December 6, 2011.
ClotCare Web site. Table on Vitamin K Content of Selected Foods. Available at
www.clotcare.org/vitaminkandwarfarin.aspx. Accessed December 16, 2011.

I. Disease Interactions: May result in increased INR and a lower warfarin dose requirement
1. Malnourished/nothing by mouth
2. Recent major surgery
3. Chronic heart failure (especially acutely decompensated)
4. Liver disease
5. Hyperthyroid state (increased clearance of clotting factors) (opposite occurs in hypothyroid state)
6. Prolonged fever (increased clearance of clotting factors)
7. Diarrhea

J. Monitoring Parameters
1. Signs and symptoms of bleeding
a. Mild: Nosebleeds, bleeding gums, easy bruising
b. More severe (evaluation needed): Hematuria, hematemesis, hemoptysis, melena, hematochezia,
bright red blood per rectum
2. INR = (PTpatient/PTmean)ISI, where PT = prothrombin time and ISI is international sensitivity index.
a. In general, check INR within 1–2 weeks after dose adjustment and every 2 weeks until 2–3
consecutive therapeutic INRs; then testing can be done less frequently.

ACCP Updates in Therapeutics® 2012: The Pharmacotherapy Preparatory Review and Recertification Course

1-266
Ambulatory Care

b. Previous CHEST guidelines recommended minimum INR testing frequency of every 4 weeks
(8th edition)
c. Latest CHEST guidelines (9th edition) state that if INR is consistently stable, suggest INR testing
frequency of up to 12 weeks (2B)

K. Home INR Monitoring and Home Warfarin Management


1. CHEST guidelines suggest patient self-testing (PST) and patient self-management (PSM) over usual
outpatient monitoring in motivated patients who show competency (2B). Recent meta-analysis of 22
trials in 8413 patients comparing PST and/or PSM with usual care (Bloomfield HE, et al. Ann Intern
Med 2011;154:472–82).
a. Results showed that patients assigned to PST/PSM had significantly lower mortality, significantly
lower risk of major thromboembolism, and no increased risk of major bleeding. NNT to prevent
one thromboembolic event: 67; NNT to prevent one death: 36
b. Many limitations, including that it was a highly selected group of very motivated adults; in half
of the trials, less than 50% of patients successfully completed training. Withdrawal rates were up
to 25%, benefit was mostly from PSM rather than PST, and strength of evidence for mortality was
low.
2. If patients are not candidates for PST and PSM, oral anticoagulation should be managed in a systematic
and coordinated fashion, with patient education, systematic INR testing, tracking, dosing, follow-up,
and communication to patients.

L. Anticoagulation During Pregnancy and Breastfeeding


1. Warfarin: Pregnancy category X
a. If a woman receiving anticoagulation for VTE becomes pregnant, she should be switched to
LMWH (not UFH). Avoid dabigatran, rivaroxaban, and apixaban.
b. If a woman with a mechanical heart valve becomes pregnant:
i. Use adjusted-dose twice-daily LMWH or UFH throughout pregnancy OR
ii. Use adjusted-dose twice-daily LMWH or UFH until 13th week of pregnancy; then switch to
warfarin; then switch back to LMWH or UFH close to delivery.
c. If a pregnant woman has an acute VTE, anticoagulants should be continued for at least 6 weeks
postpartum (for a minimum total therapy duration of 3 months).
2. Warfarin, LMWH, or UFH can be safely used while a woman is breastfeeding; use these rather than
dabigatran, rivaroxaban, or apixaban.

M. Management of Oral Anticoagulation During Invasive Procedures


1. Discontinue warfarin around 5 days before surgery (1C), and perform the procedure when the INR has
normalized.
2. Test INR on day before surgery, if feasible. If INR is still elevated (1.5 or higher), could administer
low-dose (1–2 mg) oral vitamin K
3. Give last dose of LMWH 24 hours before surgery (2C) or intravenous UFH 4–6 hours before surgery
(2C). If once-daily LMWH is used, consider using half the dose for the last dose before surgery.
4. Resume warfarin 12–24 hours after surgery (2C). Resume subcutaneous LMWH (if using) and
continue until warfarin is therapeutic; restart LMWH in 24 hours after minor surgery and in 48–72
hours after major surgery/high bleeding risk.
5. LMWH is preferred over intravenous UFH for bridging anticoagulation. Subcutaneous UFH was only
studied in a few patients for bridging anticoagulation.
6. All bridging anticoagulation with LMWH is now full therapeutic dose; no more options for
prophylactic/low-dose LMWH for bridging

ACCP Updates in Therapeutics® 2012: The Pharmacotherapy Preparatory Review and Recertification Course

1-267
Ambulatory Care

a. Bridging-dose subcutaneous LMWH:


i. Enoxaparin 1 mg/kg 2 times/day (1 mg/kg once daily if CrCl less than 30 mL/minute)
ii. Enoxaparin 1.5 mg/kg once daily
iii. Dalteparin 200 IU/kg once daily
iv. Dalteparin 100 IU/kg 2 times/day
v. Tinzaparin 175 IU/kg once daily
7. Decision to bridge is based on patient’s thromboembolic risk.
a. High thromboembolism risk (greater than 10% the annual risk of thromboembolism): BRIDGE
i. Mechanical heart valve: Any mechanical mitral valve, older aortic valve (caged-ball or tilting
disk), or recent stroke/TIA (past 6 months)
ii. Atrial fibrillation: CHADS2 score of 5 or 6, recent stroke/TIA (past 3 months), or rheumatic
valvular heart disease
iii. Venous thromboembolism: Recent VTE (past 3 months) or severe thrombophilia (protein C,
protein S or antithrombin deficiency, antiphospholipid antibodies, or multiple abnormalities)
b. Moderate thromboembolism risk (5%–10% the annual risk of thromboembolism): Decision to
bridge or not to bridge is based on individual patient risk factors.
i. Mechanical heart valve: Bileaflet aortic valve and one or more risk factor (atrial fibrillation,
stroke/TIA, hypertension, diabetes, CHF, age older than 75)
ii. Atrial fibrillation: CHADS2 score of 3 or 4
iii. Venous thromboembolism: VTE in the past 3–12 months, nonsevere thrombophilic conditions
(heterozygous factor V Leiden mutation, prothrombin gene mutation), recurrent VTE, or
active cancer (treated in past 6 months or palliative)
c. Low thromboembolism risk (less than 5% the annual risk of thromboembolism): DO NOT
BRIDGE.
i. Mechanical heart valve: Bileaflet aortic valve without atrial fibrillation or other risk factors for
stroke
ii. Atrial fibrillation: CHADS2 score of 0–2 and no previous stroke/TIA
iii. Venous thromboembolism: VTE more than 12 months ago with no risk factors
8. For most minor dental procedures (single- or multi-tooth extractions or root canal), suggest continuing
VKA, using oral pro-hemostatic agent (aminocaproic or tranexamic acid mouthwash) or discontinuing
VKA 2–3 days before procedure (2C). For cataract removal or minor dermatologic procedures, suggest
continuing VKA and using local measures (2C).

N. LMWH in Special Populations


1. Patients with severe chronic kidney disease (CKD) (CrCl less than 30 mL/minute):
a. Enoxaparin is the LMWH best studied in CKD; preferred agent
b. If LMWH is used in patients who require LMWH, a reduction in the dose is recommended
(2C). Recommended to use 50% of the usual dose of enoxaparin (e.g., 1 mg/kg once daily for
therapeutic dose)
c. For prophylaxis, recommended enoxaparin dose is 30 mg once daily
d. Dosing uncertainties are avoided if UFH is used instead of LMWH.
e. Consider antifactor Xa–level monitoring; check peak level 4 hours after dose.
i. Target anti-Xa range for once-daily enoxaparin: 1.3
ii. Target anti-Xa range for twice-daily enoxaparin: 0.6–1.0
2. Obese patients:
a. LMWH weight-based dosing should be used, based on actual weight.
b. Has been suggested in patients with a body mass index greater than 40 kg/m2 to consider
monitoring antifactor Xa levels

ACCP Updates in Therapeutics® 2012: The Pharmacotherapy Preparatory Review and Recertification Course

1-268
Ambulatory Care

c. Prophylactic enoxaparin dose in obesity is controversial: it has been suggested to consider


either giving 0.5 mg/kg subcutaneously every 12 hours or increasing the standard prophylaxis
dose by 25%.
d. Fondaparinux in obese patients (body weight more than 100 kg): dose at 10 mg subcutaneously
daily rather than the usual 7.5 mg/day (2C).

O. Prevention of VTE in Orthopedic Surgery Patients


1. Total hip arthroplasty or total knee arthroplasty: LMWH, fondaparinux, apixaban, dabigatran,
rivaroxaban, low-dose UFH, adjusted-dose VKA, or aspirin (all 1B) or an intermittent pneumatic
compression device (IPCD) (1C) are recommended for a minimum of 10–14 days. LMWH preferred
over other agents. If patients decline injections, apixaban or dabigatran (1B) are recommended over
other agents. Rivaroxaban or VKA are recommended next.
2. Hip fracture surgery: LMWH, fondaparinux, low-dose UFH, adjusted-dose VKA, or aspirin (all
1B) or an IPCD (1C) are recommended for a minimum of 10–14 days. LMWH preferred over other
agents. If patients decline injections, apixaban or dabigatran (1B) are recommended over other agents.
Rivaroxaban or VKA is recommended next.
3. In patients undergoing major orthopedic surgery, suggest extending thromboprophylaxis in the
outpatient period for up to 35 days from day of surgery over 10–14 days (2B)
4. Goal INR is 2–3 for all indications except when higher risk warrants a higher goal of 2.5–3.5;
therefore, the goal is 2–3 in orthopedic surgery patients.

P. Newer Anticoagulants
1. Dabigatran (Pradaxa)
a. FDA approved
b. Oral direct thrombin inhibitor
c. Indicated for the prevention of stroke and systemic embolism in patients with nonvalvular atrial
fibrillation
d. A 150-mg dose 2 times/day was superior to warfarin in preventing stroke/peripheral embolic
events in atrial fibrillation by 34% while reducing the risk of hemorrhagic stroke by 74%, with
comparable major bleeding rates; higher gastrointestinal bleeding (Connolly SJ, et al. RE-LY trial.
N Engl J Med 2009;361:1139–51).
e. Dose: 150 mg orally 2 times/day (if CrCl 15–30 mL/minute: 75 mg 2 times/day). Assess renal
function in all patients before starting therapy and during therapy as clinically indicated.
f. If possible, discontinue dabigatran 1–2 days (CrCl of 50 mL/minute or more) or 3–5 days
(CrCl less than 50 mL/minute) before invasive or surgical procedures. Longer times should be
considered for major or spinal/epidural procedures.
g. Contraindications: Active pathologic bleeding; history of serious hypersensitivity to dabigatran
h. Drug interactions:
i. Avoid using dabigatran with P-glycoprotein (P-gp) inducers (e.g., rifampin), which reduce
exposure to dabigatran.
ii. Concomitant P-gp inhibitors: Consider reducing dose to 75 mg 2 times/day if concomitant
ketoconazole or dronedarone and CrCl 30–50. Avoid dabigatran with CrCl less than 30 and
P-gp inhibitors above plus verapamil, amiodarone, clarithromycin, and quinidine.
i. Important patient education point: Dabigatran dispensed in bottles of 60 expires 4 months (was
previously 30 days) after opening. The change in expiration date labeling was made in November
2011. The capsules contain small pellets, which are affected by humidity. Capsules must be stored
in their original container, not placed in pillboxes or a dispenser. This is not true of dabigatran
dispensed in blister packs.

ACCP Updates in Therapeutics® 2012: The Pharmacotherapy Preparatory Review and Recertification Course

1-269
Ambulatory Care

2. Rivaroxaban (Xarelto)
a. FDA-approved oral direct factor Xa inhibitor
b. Indicated for the prophylaxis of VTE in patients undergoing total hip or knee replacement surgery.
Clinical studies show superiority to enoxaparin with no difference in major bleeding events.
i. Dose: 10 mg orally once daily; starting at least 6–10 hours after surgery
ii. Duration: 35 days for hip replacement; 12 days for knee replacement
c. Indicated for the reduction of risk of stroke and systemic embolism in nonvalvular atrial
fibrillation. Dose: 20 mg orally once daily with evening meal; 15 mg once daily if CrCl 15–20 mL/
minute; avoid if CrCl less than 15 mL/minute
d. Contraindications: Active major bleeding, hypersensitivity, severe CKD, moderate-severe hepatic
impairment, or any hepatic disease associated with coagulopathy
e. Warnings and precautions:
i. Bleeding
ii. Moderate CKD (from CrCl 30 to less than 50 mL/minute): Closely observe for signs of
bleeding.
iii Spinal or epidural anesthesia or puncture: Patients are at risk of developing spinal or epidural
hematoma/paralysis. An epidural catheter should not be removed less than 18 hours after a
dose of rivaroxaban; the next dose of rivaroxaban should not be given less than 6 hours after
removal of the catheter.
f. Drug interactions:
i. Avoid concomitant use with combined P-gp and strong CYP 3A4 inhibitors (e.g.,
ketoconazole, itraconazole, lopinavir/ritonavir, ritonavir, indinavir/ritonavir, conivaptan)
except in VTE prophylaxis if clinical data suggest an increase in exposure is unlikely to
affect bleeding risk (e.g., clarithromycin and erythromycin, which have a lesser effect than
ketoconazole and ritonavir).
ii. Avoid concomitant use with combined P-gp and strong CYP3A4 inducers (e.g.,
carbamazepine, phenytoin, rifampin, St. John’s wort).
iii. Use in patients with CrCl 15–50 mL/minute receiving concomitant combined P-gp and
weak or moderate CYP3A4 inhibitors (e.g., amiodarone, diltiazem, verapamil, quinidine,
ranolazine, dronedarone, felodipine, erythromycin, azithromycin) only if potential benefit
justifies potential risk of increased rivaroxaban exposure.
3. Apixaban (Eliquis)
a. Investigational oral direct factor Xa inhibitor; expected to gain FDA approval by March 2012
b. Shown to be non-inferior to warfarin for stroke prevention in atrial fibrillation
4. Desirudin (Iprivask)
a. FDA approved
b. Specific inhibitor of human thrombin
c. Structure similar to hirudin, the naturally occurring anticoagulant in the peripharyngeal glands of
the medicinal leech
d. Indicated for the prophylaxis of DVT in patients undergoing elective hip replacement surgery
e. Dose: 15 mg subcutaneously every 12 hours; give initial dose 5–15 minutes before surgery.
Duration of up to 12 days has been well tolerated.
f. Renal impairment: If CrCl is 31–60 mL/minute: Initial dose 5 mg every 12 hours. If CrCl is less
than 31 mL/minute: Initial dose 1.7 mg every 12 hours
i. Use with caution in patients with renal impairment (CrCl of 60 mL/minute or less).
ii. Daily activated partial thromboplastin time (aPTT) and daily serum creatinine monitoring are
recommended in these patients; reduce dose if peak aPTT level exceeds 2 times control (after
holding until aPTT returns to less than 2 times control).

ACCP Updates in Therapeutics® 2012: The Pharmacotherapy Preparatory Review and Recertification Course

1-270
Ambulatory Care
QuickGuide

IV. ADULT IMMUNIZATIONS


FIGURE 1. Recommended adult immunization schedule, by vaccine and age group1 — United States, 2012
QuickGuide
VACCINE AGE GROUP 19–21 years 22–26 years 27–49 years 50–59 years 60–64 years ≥65 years

Influenza2,* 1 dose annually


FIGURE 1—
Tetanus,1. Recommended
diphtheria, pertussisadult immunization
(Td/Tdap)3,* schedule, by vaccine
Substitute 1-time dose ofand
Tdapage
for Tdgroup
booster; United
then States,
boost with 2012
Td every 10 years Td/Tdap3

Varicella 4, 2 doses
VACCINE * AGE GROUP 19–21 years 22–26 years 27–49 years 50–59 years 60–64 years ≥65 years
Human papillomavirus (HPV) 5,* Female 3 doses
Influenza *2, 1 dose annually
Human papillomavirus (HPV) 5,* Male 3 doses
Tetanus, diphtheria, pertussis (Td/Tdap)3,* Substitute 1-time dose of Tdap for Td booster; then boost with Td every 10 years Td/Tdap3
Zoster6 1 dose
Varicella4,* 2 doses
Measles, mumps, rubella (MMR)7,* 1 or 2 doses 1 dose
Human papillomavirus (HPV) 5,* Female 3 doses
Pneumococcal (polysaccharide)8,9 1 or 2 doses 1 dose
Human papillomavirus (HPV) 5,* Male 3 doses
Meningococcal10,* 1 or more doses
Zoster6 1 dose
Hepatitis A11,* 2 doses
Measles, mumps, rubella (MMR)7,* 1 or 2 doses 1 dose
Hepatitis B12,* 3 doses
Pneumococcal (polysaccharide)8,9 1 or 2 doses 1 dose
* Covered by the Vaccine Injury Compensation Program
Meningococcal10,* 1 or more doses
For all persons in this category Recommended if some Tdap recommended for ≥65 No recommendation
A11,*meet the age requirements
Hepatitiswho other risk factor is present 2 doses
if contact with <12 month
and who lack documentation of (e.g., on the basis of medical, old child. Either Td or Tdap
B12,*
Hepatitisvaccination or have no evidence occupational, lifestyle, can be3used
doses
if no infant
* Covered byof previous
the Vaccineinfection
Injury Compensation Program or other indications) contact

For all persons in this category Recommended if some Tdap recommended for ≥65 No recommendation
who meet the age requirements other risk factor is present if contact with <12 month
FIGURE 2.and Vaccines that might be
who lack documentation of indicated for adults,
(e.g., on the based on medical and otheroldindications
basis of medical, 1
child. Either Td— United
or Tdap States, 2012
Figure 1. Recommended adult immunization schedule, by vaccine and age group — United States, 2012
vaccination or have no evidence occupational, lifestyle, can be used if no infant
1
of previous infection or other indications)
HIV infection 4, 7, 13, 14 contact Asplenia 13
INDICATION
Immunocom- CD4+ (including Diabetes,
promising T lymphocyte count elective kidney
conditions (ex- Heart disease, splenectomy failure, end-
FIGURE 2. Vaccines that might be indicated cluding human for adults, based on medical and other indications
chronic lung 1and
— United
persistentStates, 2012 stage renal
immunode- <200 Men who disease, complement Chronic disease, Health-
INDICATION ficiency virus cells/
HIV infection≥200
4, 7, 13, 14 have sex with chronic component
Asplenia13 liver receipt of care
VACCINE Pregnancy [HIV])4,6,7,14
Immunocom- µL CD4cells/µL
+ men (MSM) alcoholism deficiencies)
(including disease hemodialysis
Diabetes, personnel
promising T lymphocyte count 1 dose TIV or
elective kidney 1 dose TIV or
Influenza2,* 1 dose TIV(ex-
conditions annually LAIV annually Heart disease, 1 dose TIV annually
splenectomy failure, end- LAIV annually
cluding human chronic lung and persistent stage renal
Tetanus, diphtheria, per- immunode- <200 Men who disease, complement Chronic disease, Health-
tussis (Td/Tdap)3,* Substitute 1-time dose of Tdap for Td booster; then boost with Td every 10 years
ficiency virus cells/ ≥200 have sex with chronic component liver receipt of care
VACCINE
Varicella 4,* Pregnancy [HIV])4,6,7,14
Contraindicated µL cells/µL men (MSM) alcoholism deficiencies) disease hemodialysis personnel
2 doses
1 dose TIV or 1 dose TIV or
Influenza 2,*
Human papillomavirus 1 dose TIV annually LAIV annually 1 dose TIV annually LAIV annually
(HPV)5,* Female 3 doses through age 26 years 3 doses through age 26 years
Tetanus, diphtheria, per-
Human
tussis papillomavirus
(Td/Tdap) 3,* Substitute 1-time dose of Tdap for Td booster; then boost with Td every 10 years
(HPV)5,* Male 3 doses through age 26 years 3 doses through age 21 years
Varicella4,* Contraindicated 2 doses
Zoster6 Contraindicated 1 dose
Human papillomavirus
Measles,
(HPV) mumps, rubella7,*
5,* Female 3 doses through age 26 years
Contraindicated 1 or32doses
dosesthrough age 26 years
Pneumococcal
Human papillomavirus
(HPV)5,* Male 8,9
(polysaccharide) 3 doses through age 26 years 1 or 2 doses 3 doses through age 21 years
Meningococcal
Zoster 6 10,*
Contraindicated 1 or more doses 1 dose
Hepatitis A11,* rubella7,*
Measles, mumps, Contraindicated 2 doses 1 or 2 doses
Hepatitis B12,*
Pneumococcal 3 doses
(polysaccharide)8,9 1 or 2 doses
* Covered by the Vaccine Injury Compensation Program
Meningococcal10,* 1 or more doses
For all persons in this category Recommended if some Contraindicated No recommendation
Hepatitis Awho
11,* meet the age requirements other risk factor is present 2 doses
and who lack documentation of (e.g., on the basis of medical,
12,
Hepatitis Bvaccination
* or have no evidence occupational, lifestyle, 3 doses
of previous infection or other indications)
* Covered by the Vaccine Injury Compensation Program
For all persons in this category Recommended if some Contraindicated No recommendation
who meet the age requirements other risk factor is present
and who lack documentation of (e.g., on the basis of medical,
vaccination orNOTE:
have noThe above recommendations
evidence occupational,must be read along with the footnotes on pages 4–5 of this schedule.
lifestyle,
of previous infection or other indications)

Figure 2. Vaccines that might be indicated for adults, based on medical and other indications1 — United States, 2012
NOTE: The above recommendations must be read along with the footnotes
MMWR on pages 4–5
/ February of this
3, 2012 schedule.
/ Vol. 61 / No. 4 3
Note: The above recommendations must be read along with the footnotes on the following pages of this schedule.

MMWR / February 3, 2012 / Vol. 61 / No. 4 3

ACCP Updates in Therapeutics® 2012: The Pharmacotherapy Preparatory Review and Recertification Course

1-271
Ambulatory Care
QuickGuide

1. Additional information • HPV vaccines are not live vaccines and can be administered to persons who are
• Advisory Committee on Immunization Practices (ACIP) vaccine recommendations and immunocompromised as a result of infection (including HIV infection), disease, or
additional information are available at: http://www.cdc.gov/vaccines/pubs/acip-list.htm. medications. Vaccine is recommended for immunocompromised persons through
• Information on travel vaccine requirements and recommendations (e.g., for hepatitis A age 26 years who did not get any or all doses when they were younger. The immune
and B, meningococcal, and other vaccines) available at http://wwwnc.cdc.gov/travel/ response and vaccine efficacy might be less than that in immunocompetent persons.
page/vaccinations.htm. • Men who have sex with men (MSM) might especially benefit from vaccination to
2. Influenza vaccination prevent condyloma and anal cancer. HPV4 is recommended for MSM through age 26
• Annual vaccination against influenza is recommended for all persons 6 months of age years who did not get any or all doses when they were younger.
and older. • Ideally, vaccine should be administered before potential exposure to HPV through
• Persons 6 months of age and older, including pregnant women, can receive the trivalent sexual activity; however, persons who are sexually active should still be vaccinated
inactivated vaccine (TIV). consistent with age-based recommendations. HPV vaccine can be administered to
• Healthy, nonpregnant adults younger than age 50 years without high-risk medical persons with a history of genital warts, abnormal Papanicolaou test, or positive HPV
conditions can receive either intranasally administered live, attenuated influenza DNA test.
vaccine (LAIV) (FluMist), or TIV. Health-care personnel who care for severely • A complete series for either HPV4 or HPV2 consists of 3 doses. The second dose should
immunocompromised persons (i.e., those who require care in a protected environment) be administered 1–2 months after the first dose; the third dose should be administered
should receive TIV rather than LAIV. Other persons should receive TIV. 6 months after the first dose (at least 24 weeks after the first dose).
• The intramuscular or intradermal administered TIV are options for adults aged 18–64 • Although HPV vaccination is not specifically recommended for health-care personnel
years. (HCP) based on their occupation, HCP should receive the HPV vaccine if they are in the
• Adults aged 65 years and older can receive the standard dose TIV or the high-dose TIV recommended age group.
(Fluzone High-Dose). 6. Zoster vaccination
3. Tetanus, diphtheria, and acellular pertussis (Td/Tdap) vaccination • A single dose of zoster vaccine is recommended for adults 60 years of age and older
• Administer a one-time dose of Tdap to adults younger than age 65 years who have not regardless of whether they report a prior episode of herpes zoster. Although the
received Tdap previously or for whom vaccine status is unknown to replace one of the vaccine is licensed by the Food and Drug Administration (FDA) for use among and
10-year Td boosters. can be administered to persons 50 years and older, ACIP recommends that vaccination
• Tdap is specifically recommended for the following persons: begins at 60 years of age.
— pregnant women more than 20 weeks’ gestation, • Persons with chronic medical conditions may be vaccinated unless their condition
— adults, regardless of age, who are close contacts of infants younger than age 12 constitutes a contraindication, such as pregnancy or severe immunodeficiency.
months (e.g., parents, grandparents, or child care providers), and • Although zoster vaccination is not specifically recommended for health-care personnel
— health-care personnel. (HCP), HCP should receive the vaccine if they are in the recommended age group.
• Tdap can be administered regardless of interval since the most recent tetanus or 7. Measles, mumps, rubella (MMR) vaccination
diphtheria-containing vaccine. • Adults born before 1957 generally are considered immune to measles and mumps. All
• Pregnant women not vaccinated during pregnancy should receive Tdap immediately adults born in 1957 or later should have documentation of 1 or more doses of MMR
postpartum. vaccine unless they have a medical contraindication to the vaccine, laboratory evidence
• Adults 65 years and older may receive Tdap. of immunity to each of the three diseases, or documentation of provider-diagnosed
• Adults with unknown or incomplete history of completing a 3-dose primary vaccination measles or mumps disease. For rubella, documentation of provider-diagnosed disease
series with Td-containing vaccines should begin or complete a primary vaccination is not considered acceptable evidence of immunity.
series. Tdap should be substituted for a single dose of Td in the vaccination series with Measles component:
Tdap preferred as the first dose. • A routine second dose of MMR vaccine, administered a minimum of 28 days after the
• For unvaccinated adults, administer the first 2 doses at least 4 weeks apart and the first dose, is recommended for adults who
third dose 6–12 months after the second. — are students in postsecondary educational institutions;
• If incompletely vaccinated (i.e., less than 3 doses), administer remaining doses. — work in a health-care facility; or
Refer to the ACIP statement for recommendations for administering Td/Tdap as — plan to travel internationally.
prophylaxis in wound management (See footnote 1). • Persons who received inactivated (killed) measles vaccine or measles vaccine of
4. Varicella vaccination unknown type from 1963 to 1967 should be revaccinated with 2 doses of MMR vaccine.
• All adults without evidence of immunity to varicella (as defined below) should receive Mumps component:
2 doses of single-antigen varicella vaccine or a second dose if they have received only • A routine second dose of MMR vaccine, administered a minimum of 28 days after the
1 dose. first dose, is recommended for adults who
• Special consideration for vaccination should be given to those who — are students in postsecondary educational institutions;
— have close contact with persons at high risk for severe disease (e.g., health-care — work in a health-care facility; or
personnel and family contacts of persons with immunocompromising condi- — plan to travel internationally.
tions) or • Persons vaccinated before 1979 with either killed mumps vaccine or mumps vaccine of
— are at high risk for exposure or transmission (e.g., teachers; child care employees; unknown type who are at high risk for mumps infection (e.g., persons who are working
residents and staff members of institutional settings, including correctional in a health-care facility) should be considered for revaccination with 2 doses of MMR
institutions; college students; military personnel; adolescents and adults living vaccine.
in households with children; nonpregnant women of childbearing age; and Rubella component:
international travelers). • For women of childbearing age, regardless of birth year, rubella immunity should be
• Pregnant women should be assessed for evidence of varicella immunity. Women who determined. If there is no evidence of immunity, women who are not pregnant should
do not have evidence of immunity should receive the first dose of varicella vaccine be vaccinated. Pregnant women who do not have evidence of immunity should receive
upon completion or termination of pregnancy and before discharge from the health- MMR vaccine upon completion or termination of pregnancy and before discharge from
care facility. The second dose should be administered 4–8 weeks after the first dose. the health-care facility.
• Evidence of immunity to varicella in adults includes any of the following: Health-care personnel born before 1957:
— documentation of 2 doses of varicella vaccine at least 4 weeks apart; • For unvaccinated health-care personnel born before 1957 who lack laboratory evidence
— U.S.-born before 1980 (although for health-care personnel and pregnant women, of measles, mumps, and/or rubella immunity or laboratory confirmation of disease,
birth before 1980 should not be considered evidence of immunity); health-care facilities should consider routinely vaccinating personnel with 2 doses of
— history of varicella based on diagnosis or verification of varicella by a health-care MMR vaccine at the appropriate interval for measles and mumps or 1 dose of MMR
provider (for a patient reporting a history of or having an atypical case, a mild vaccine for rubella.
case, or both, health-care providers should seek either an epidemiologic link to a 8. Pneumococcal polysaccharide (PPSV) vaccination
typical varicella case or to a laboratory-confirmed case or evidence of laboratory • Vaccinate all persons with the following indications:
confirmation, if it was performed at the time of acute disease); — age 65 years and older without a history of PPSV vaccination;
— history of herpes zoster based on diagnosis or verification of herpes zoster by a — adults younger than 65 years with chronic lung disease (including chronic
health-care provider; or obstructive pulmonary disease, emphysema, and asthma); chronic cardiovascular
— laboratory evidence of immunity or laboratory confirmation of disease. diseases; diabetes mellitus; chronic liver disease (including cirrhosis); alcoholism;
5. Human papillomavirus (HPV) vaccination cochlear implants; cerebrospinal fluid leaks; immunocompromising conditions;
• Two vaccines are licensed for use in females, bivalent HPV vaccine (HPV2) and and functional or anatomic asplenia (e.g., sickle cell disease and other hemoglo-
quadrivalent HPV vaccine (HPV4), and one HPV vaccine for use in males (HPV4). binopathies, congenital or acquired asplenia, splenic dysfunction, or splenectomy
• For females, either HPV4 or HPV2 is recommended in a 3-dose series for routine [if elective splenectomy is planned, vaccinate at least 2 weeks before surgery]);
vaccination at 11 or 12 years of age, and for those 13 through 26 years of age, if not — residents of nursing homes or long-term care facilities; and
previously vaccinated. — adults who smoke cigarettes.
• For males, HPV4 is recommended in a 3-dose series for routine vaccination at 11 or 12 • Persons with asymptomatic or symptomatic HIV infection should be vaccinated as
years of age, and for those 13 through 21 years of age, if not previously vaccinated. soon as possible after their diagnosis.
Males 22 through 26 years of age may be vaccinated.

4 MMWR / February 3, 2012 / Vol. 61 / No. 4

ACCP Updates in Therapeutics® 2012: The Pharmacotherapy Preparatory Review and Recertification Course

1-272
Ambulatory Care
QuickGuide

• When cancer chemotherapy or other immunosuppressive therapy is being considered, • Single-antigen vaccine formulations should be administered in a 2-dose schedule at
the interval between vaccination and initiation of immunosuppressive therapy should either 0 and 6–12 months (Havrix), or 0 and 6–18 months (Vaqta). If the combined
be at least 2 weeks. Vaccination during chemotherapy or radiation therapy should be hepatitis A and hepatitis B vaccine (Twinrix) is used, administer 3 doses at 0, 1, and 6
avoided. months; alternatively, a 4-dose schedule may be used, administered on days 0, 7, and
• Routine use of PPSV is not recommended for American Indians/Alaska Natives or 21–30 followed by a booster dose at month 12.
other persons younger than 65 years of age unless they have underlying medical 12. Hepatitis B vaccination
conditions that are PPSV indications. However, public health authorities may consider • Vaccinate persons with any of the following indications and any person seeking
recommending PPSV for American Indians/Alaska Natives who are living in areas where protection from hepatitis B virus (HBV) infection:
the risk for invasive pneumococcal disease is increased. — sexually active persons who are not in a long-term, mutually monogamous
9. Revaccination with PPSV relationship (e.g., persons with more than one sex partner during the previous
• One-time revaccination 5 years after the first dose is recommended for persons 19 6 months); persons seeking evaluation or treatment for a sexually transmitted
through 64 years of age with chronic renal failure or nephrotic syndrome; functional disease (STD); current or recent injection-drug users; and men who have sex
or anatomic asplenia (e.g., sickle cell disease or splenectomy); and for persons with with men;
immunocompromising conditions. — health-care personnel and public-safety workers who are exposed to blood or
• Persons who received PPSV before age 65 years for any indication should receive other potentially infectious body fluids;
another dose of the vaccine at age 65 years or later if at least 5 years have passed since — persons with diabetes younger than 60 years as soon as feasible after diagnosis;
their previous dose. persons with diabetes who are 60 years or older at the discretion of the treating
• No further doses are needed for persons vaccinated with PPSV at or after age 65 years. clinician based on increased need for assisted blood glucose monitoring in long-
10. Meningococcal vaccination term care facilities, likelihood of acquiring hepatitis B infection, its complications
• Administer 2 doses of meningococcal conjugate vaccine quadrivalent (MCV4) at least 2 or chronic sequelae, and likelihood of immune response to vaccination;
months apart to adults with functional asplenia or persistent complement component — persons with end-stage renal disease, including patients receiving hemodialysis;
deficiencies. persons with HIV infection; and persons with chronic liver disease;
• HIV-infected persons who are vaccinated should also receive 2 doses. — household contacts and sex partners of persons with chronic HBV infection; clients
• Administer a single dose of meningococcal vaccine to microbiologists routinely exposed and staff members of institutions for persons with developmental disabilities;
to isolates of Neisseria meningitidis, military recruits, and persons who travel to or live and international travelers to countries with high or intermediate prevalence of
in countries in which meningococcal disease is hyperendemic or epidemic. chronic HBV infection; and
• First-year college students up through age 21 years who are living in residence halls — all adults in the following settings: STD treatment facilities; HIV testing and treat-
should be vaccinated if they have not received a dose on or after their 16th birthday. ment facilities; facilities providing drug-abuse treatment and prevention services;
• MCV4 is preferred for adults with any of the preceding indications who are 55 years old health-care settings targeting services to injection-drug users or men who have
and younger; meningococcal polysaccharide vaccine (MPSV4) is preferred for adults sex with men; correctional facilities; end-stage renal disease programs and facili-
56 years and older. ties for chronic hemodialysis patients; and institutions and nonresidential daycare
• Revaccination with MCV4 every 5 years is recommended for adults previously facilities for persons with developmental disabilities.
vaccinated with MCV4 or MPSV4 who remain at increased risk for infection (e.g., • Administer missing doses to complete a 3-dose series of hepatitis B vaccine to those
adults with anatomic or functional asplenia or persistent complement component persons not vaccinated or not completely vaccinated. The second dose should be
deficiencies). administered 1 month after the first dose; the third dose should be given at least 2
11. Hepatitis A vaccination months after the second dose (and at least 4 months after the first dose). If the combined
• Vaccinate any person seeking protection from hepatitis A virus (HAV) infection and hepatitis A and hepatitis B vaccine (Twinrix) is used, give 3 doses at 0, 1, and 6 months;
persons with any of the following indications: alternatively, a 4-dose Twinrix schedule, administered on days 0, 7, and 21–30 followed
— men who have sex with men and persons who use injection drugs; by a booster dose at month 12 may be used.
— persons working with HAV-infected primates or with HAV in a research labora- • Adult patients receiving hemodialysis or with other immunocompromising conditions
tory setting; should receive 1 dose of 40 µg/mL (Recombivax HB) administered on a 3-dose schedule
— persons with chronic liver disease and persons who receive clotting factor or 2 doses of 20 µg/mL (Engerix-B) administered simultaneously on a 4-dose schedule
concentrates; at 0, 1, 2, and 6 months.
— persons traveling to or working in countries that have high or intermediate 13. Selected conditions for which Haemophilus influenzae type b (Hib) vaccine may be used
endemicity of hepatitis A; and • 1 dose of Hib vaccine should be considered for persons who have sickle cell disease,
— unvaccinated persons who anticipate close personal contact (e.g., household or leukemia, or HIV infection, or who have anatomic or functional asplenia if they have
regular babysitting) with an international adoptee during the first 60 days after not previously received Hib vaccine.
arrival in the United States from a country with high or intermediate endemic- 14. Immunocompromising conditions
ity. (See footnote 1 for more information on travel recommendations). The first • Inactivated vaccines generally are acceptable (e.g., pneumococcal, meningococcal, and
dose of the 2-dose hepatitis A vaccine series should be administered as soon as influenza [inactivated influenza vaccine]), and live vaccines generally are avoided in
adoption is planned, ideally 2 or more weeks before the arrival of the adoptee. persons with immune deficiencies or immunocompromising conditions. Information
on specific conditions is available at http://www.cdc.gov/vaccines/pubs/acip-list.htm.

These schedules indicate the recommended age groups and medical indications for which administration of currently licensed vaccines is commonly indicated for adults ages
19 years and older, as of January 1, 2012. For all vaccines being recommended on the adult immunization schedule: a vaccine series does not need to be restarted, regardless of
the time that has elapsed between doses. Licensed combination vaccines may be used whenever any components of the combination are indicated and when the vaccine’s other
components are not contraindicated. For detailed recommendations on all vaccines, including those used primarily for travelers or that are issued during the year, consult the manu-
facturers’ package inserts and the complete statements from the Advisory Committee on Immunization Practices (http:// www.cdc.gov/vaccines/pubs/acip-list.htm).
Report all clinically significant postvaccination reactions to the Vaccine Adverse Event Reporting System (VAERS). Reporting forms and instructions on filing a VAERS report are avail-
able at http://www.vaers.hhs.gov or by telephone, 800-822-7967.
Information on how to file a Vaccine Injury Compensation Program claim is available at http://www.hrsa.gov/vaccinecompensation or by telephone, 800-338-2382. Information
about filing a claim for vaccine injury is available through the U.S. Court of Federal Claims, 717 Madison Place, N.W., Washington, D.C. 20005; telephone, 202-357-6400.
Additional information about the vaccines in this schedule, extent of available data, and contraindications for vaccination also is available at http://www.cdc.gov/vaccines or from
the CDC-INFO Contact Center at 800-CDC-INFO (800-232-4636) in English and Spanish, 8:00 a.m. to 8:00 p.m., Monday through Friday, excluding holidays.
Use of trade names and commercial sources is for identification only and does not imply endorsement by the U.S. Department of Health and Human Services.

U.S. Department of Health and Human Services • Centers for Disease Control and Prevention

U.S. Department of Health and Human Services (DHHS). Centers for Disease Control and Prevention (CDC). Advisory Committee on
Immunization Practices (ACIP). Recommended Adult Immunization Schedule, United States – 2012. Available at www.cdc.gov/vaccines/recs/
schedules/downloads/adult/mmwr-adult-schedule.pdf. Accessed

MMWR / February 3, 2012 / Vol. 61 / No. 4 5

ACCP Updates in Therapeutics® 2012: The Pharmacotherapy Preparatory Review and Recertification Course

1-273
Ambulatory Care

B. Current Issues with Herpes Zoster Vaccine (HZV): The HZV package insert states not to give HZV and
pneumococcal polysaccharide vaccine (PPSV) concurrently but to separate them by at least 4 weeks
because of decreased immunologic response to HZV. Their conclusion is based on a Merck-sponsored,
unpublished study. The ACIP states that the clinical relevance of this is unknown, and a subsequent
study showed no compromise in HZV efficacy. The ACIP/CDC, which reviewed the data, continues to
recommend that HZV and PPSV be administered at the same visit if the person is eligible for both vaccines.

C. Major Changes in the 2012 Adult Immunization Schedule from the 2011 Schedule:
1. Human papillomavirus (HPV) vaccine. HPV4 is now recommended for routine vaccination of males at
age 11–12, or at age 13–21 if not previously vaccinated, and for ages 22–26 in certain high-risk groups
if not previously vaccinated (HIV-positive, immunocompromised, or in men who have sex with men).
Previous schedule stated that HPV4 “may be administered” to males.
2. Hepatitis B vaccine. Hepatitis B vaccine is now recommended for all individuals younger than 60 with
diabetes, as soon as possible after diabetes is diagnosed. In individuals with diabetes aged 60 or older,
vaccination is at the clinician’s discretion, based on the patient’s likely need for assisted blood glucose
monitoring, likelihood of acquiring hepatitis B, and likelihood of immune response to vaccination.
3. Meningococcal vaccine. For first-year college students, specific ages for vaccination were added: those
through age 21 years who are living in residence halls should be vaccinated if they have not received a
dose on or after their 16th birthday.

Patient Cases
22. E.V. is a 71-year-old woman with COPD. Her only drug is tiotropium (Spiriva) inhaled 1 capsule/day. She
received the influenza vaccine last October, her last Td vaccine was at age 65, and her pneumococcal poly-
saccharide vaccine (PPSV) was at age 60. She has a new grandson (3 months old), whom she cares for while
her daughter is at work. Which one of the following is the most appropriate choice for vaccine(s) that should
be given at her October internal medicine clinic appointment?
A. Only the influenza vaccine should be given.
B. Influenza and PPSV vaccines should be given.
C. Influenza, PPSV, and zoster vaccines should be given.
D. Influenza, PPSV, zoster, and Tdap vaccines should be given.

23. S.C., a 20-year-old woman who is going away to college, presents for a physical examination in July. She
will be living in the dormitory. She smokes ½ pack/day but has no other medical conditions. She is up to
date with all of her routine childhood vaccines, but she has not received any vaccines in the past 11 years.
She is not sexually active. Which of the following is the most appropriate choice for vaccine(s) that should
be given today?
A. Td and HPV vaccines.
B. Tdap, meningococcal (MCV4), and HPV vaccines.
C. MCV4, PPSV, and Td vaccines.
D. MCV4, PPSV, Tdap, and HPV vaccines.

ACCP Updates in Therapeutics® 2012: The Pharmacotherapy Preparatory Review and Recertification Course

1-274
Ambulatory Care

REFERENCES

Asthma 3. McEvoy CE, Neiwoehner DE. Adverse effects of


1. National Institutes of Health National Heart, Lung corticosteroid therapy for COPD: a critical review.
and Blood Institute. National Asthma Education and Chest 1997;111:732–43.
Prevention Program (NAEPP) guidelines. NAEPP 4. Fiore MC, Jaen CR, Baker TB, et al. Treating to-
Expert Panel Report 3. NIH Publication 08-5846. bacco use and dependence: 2008 update. Rock-
July 2007. Available at www.nhlbi.nih.gov/guide- ville, MD: U.S. Department of Health and Human
lines/index.htm. Accessed September 23, 2011. Services, Public Health Service, 2008. Available
2. Perera BJ. Salmeterol multicentre asthma research at www.ncbi.nlm.nih.gov/bookshelf/br.fcgi?book
trial (SMART): interim analysis shows increased =hsahcpr&part=A28163. Accessed September 26,
risk of asthma-related deaths. Ceylon Med J 2011.
2003;48:99. 5. Calverley PMA, Anderson JA, Celli B, et al. Sal-
3. Nelson HS, Weiss ST, Bleecker ER, et al. The salme- meterol and fluticasone propionate and survival
terol multicenter asthma research trial (SMART): a in chronic obstructive pulmonary disease (the
comparison of usual pharmacotherapy for asthma TORCH study). N Engl J Med 2007;356:775–89.
or usual pharmacotherapy plus salmeterol. Chest 6. Nannini LJ, Cates CJ, Lasserson TJ, Poole P.
2006;129:15–26. Combined corticosteroid and long-acting beta-
4. Agency for Healthcare Research and Quality. agonist in one inhaler versus placebo for chronic
Available at www.ahrq.gov/qual/asthmacare/asth- obstructive pulmonary disease. Cochrane Data-
mod4.htm. Accessed September 23, 2011. base Syst Rev 2007;4:CD003794.
5. Peters SP, Kunselman SJ, Icitovic N, et al. Tiotro- 7. Ernst P, Gonzalez AP, Brassard P, Suissa S. In-
pium bromide step-up therapy for adults with un- haled corticosteroid use in chronic obstructive
controlled asthma (TALC study). N Engl J Med pulmonary disease and the risk of hospitaliza-
2010;363:1715–26. tion for pneumonia. Am J Respir Crit Care Med
2007;176:162–6.
6. Martinez FD, Chinchilli VM, Morgan WJ, et al.
Use of beclomethasone dipropionate as rescue 8. Singh S, Amin AV, Loke YK. Long term use of
treatment for children with mild persistent asthma inhaled corticosteroids and the risk of pneumonia
(TREXA): a randomized, double-blind, placebo- in chronic obstructive pulmonary disease. Arch In-
controlled trial. Lancet 2011;377:650–7. tern Med 2009;169:219–29.
7. Price D, Musgrave SD, Shepstone L, et al. Leukotri- 9. Heffner JE, Mularski RA, Calverley PM. COPD
ene antagonists as first-line or add-on asthma-con- performance measures: missing opportunities for
troller therapy. N Engl J Med 2011;364:1695–707. improving care. Chest 2010;137:1181–9.
10. Rutten FH, Zuithoff NP, Hak E, Grobbee DE,
Hoes AW. Beta-blockers may reduce mortality
Chronic Obstructive Pulmonary Disease
and risk of exacerbations in patients with chronic
1. Global Initiative for Chronic Obstructive Lung obstructive pulmonary disease. Arch Intern Med
Disease Workshop Executive Summary: global 2010;170:880–7.
strategy for the diagnosis, management, and pre-
11. Salpeter SR, Ormiston T, Salpeter E, Poole P, Cates
vention of chronic obstructive pulmonary disease.
C. Cardioselective beta blockers for chronic ob-
National Institutes of Health National Heart, Lung
structive pulmonary disease. Cochrane Database
and Blood Institute, 2011 Revision. Available at
Syst Rev 2002;2:CD003566. Review. Update in:
www.goldcopd.org/. Accessed March 1, 2012.
Cochrane Database Syst Rev 2005;4:CD003566.
2. van Grunsven PM, van Schayck CP, Dereene JP,
12. Bestall JC, Paul EA, Garrod R, Garnham R, Jones
et al. Long-term effects of inhaled corticosteroids
PW, Wedzicha JA. Usefulness of the Medical Re-
in chronic obstructive pulmonary disease: a meta-
search Council (MRC) dyspnea scale as a measure
analysis. Thorax 1999;54:7–14.

ACCP Updates in Therapeutics® 2012: The Pharmacotherapy Preparatory Review and Recertification Course

1-275
Ambulatory Care

of disability in patients with chronic obstructive


pulmonary disability in patients with chronic ob-
structive pulmonary disease. Thorax 1999;54:5816.
13. VanDerMolen T, Willemse BW, Schokker S, Ten-
Hacken NH, Postma DS, Juniper EF. Develop-
ment, validity and responsiveness of the Clinical
COPD Questionnaire. Health Qual Life Outcomes
2003;1:13.
14. Welte T, Miravitlles M, Hernandez P, et al. Effica-
cy and tolerability of budesonide/formoterol added
to tiotropium in patients with chronic obstructive
pulmonary disease. Am J Respir Crit Care Med
2009;180:741–50.
15. Vogelmeier C, Hederer B, Glaab T, et al. Tiotro-
pium versus salmeterol for the prevention of exac-
erbations of COPD (POET-COPD study). N Engl J
Med 2011;364:1093–103.
16. Albert RK, Connett J, Bailey WC, et al. Azithro-
mycin for prevention of exacerbations of COPD. N
Engl J Med 2011;365:689–98.

Anticoagulation
1. Hull RD, Raskob GE, Rosenbloom D, et al. Hep-
arin for 5 days as compared with 10 days in the
initial treatment of proximal venous thrombosis. N
Engl J Med 1990;322:1260–4.
2. Guyatt GH, Akl EA, Crowther M, Gutterman DD,
Schuunemann HF. Antithrombotic Therapy and Pre-
vention of Thrombosis, 9th ed. American College
of Chest Physicians Evidence-Based Clinical Prac-
tice Guidelines. Chest 2012;141:2(Suppl):1S–70S.
3. Bloomfield HE, Krause A, Greer N, et al. Me-
ta-analysis: effect of patient self-testing and
self-management of long-term anticoagulation
on major clinical outcomes. Ann Intern Med
2011;154:472–82.

Immunizations
1. Department of Health and Human Services. Cen-
ters for Disease Control and Prevention. Recom-
mended Adult Immunization Schedule: United
States – 2012. Available at www.cdc.gov/vaccines/
recs/schedules/downloads/adult/mmwr-adult-
schedule.pdf. Accessed March 8, 2012.

ACCP Updates in Therapeutics® 2012: The Pharmacotherapy Preparatory Review and Recertification Course

1-276
Ambulatory Care

Answers and Explanations to Patient Cases

1. Answer: A as initial therapy in this age group (5–11). The BMS


The patient has symptoms 2 days/week. Intermittent dose (80 mcg 2 times/day) is low-dose ICS for this age
asthma is when the frequency of symptoms and SABA group. Montelukast is not recommended for moderate-
use are 2 days/week or less; nighttime symptoms are persistent asthma in this age group as monotherapy; it is
2 nights/month or less. Normal activity is not limited, recommended only in combination with a LABA. The
FEV1 is more than 80% of predicted, and FEV1/FVC is fluticasone/salmeterol 100/50 twice daily is medium-
normal. Because her symptoms are caused by exercise, dose ICS plus a LABA, which is step 4 in this age group.
exercise-induced asthma should be considered.
This patient requires step 3 therapy for moderate per-
2. Answer: A sistent asthma. Medium-dose ICS alone, as well as low-
Because she has mild intermittent asthma, step 1 is dose ICS plus a LABA, is an option. In light of the 2010
recommended for initial treatment. No daily medica- FDA statement about LABA safety, it may be prudent
tions are needed; an inhaled β2-agonist should be used to start with medium-dose ICS alone and then add a
as needed. Use of an inhaled β2-agonist more than 2 LABA if the patient is not controlled on ICS alone. Mo-
days/week would indicate the need to step up therapy. metasone/formoterol 200/5 mcg 2 puffs 2 times/day is
Montelukast could be recommended for step 2 (alterna- a high-dose inhaled steroid plus a LABA, which is step
tive). Mometasone would be recommended for step 2 5. Fluticasone 110 mcg/puff 1 puff 2 times/day is a low-
(first line). Omalizumab is recommended for step 5 or 6 dose steroid, which is step 2.
in patients who have allergies.
7. Answer: B
3. Answer: D In an asthma action plan, the green zone instructions
Her asthma falls into the “not well-controlled” catego- are to take the controller agent only (and the patient
ry because the frequency of her nighttime symptoms may use albuterol HFA 1–2 puffs every 4–6 hours only
is 1–3 nights/week. Recommended action for treat- if needed for periodic symptoms). In this case, the con-
ment is to go up one step to step 2: inhaled steroid (low troller is Advair 250/50 1 puff 2 times/day.
dose). Alternatives (not first line) include an LTM or
sustained-release theophylline. Budesonide-formoterol 8. Answer: B
MDI is incorrect because it is a step 3 treatment. In an asthma action plan, the yellow zone instructions
are for albuterol HFA 2–6 puffs; repeat in 20 minutes
4. Answer: A if needed, and then reassess. For a milder yellow zone
Her asthma falls into the “well-controlled” category. exacerbation, 2 puffs should be sufficient, but for a
She is using a SABA once weekly to treat symptoms. moderate exacerbation, 4–6 puffs could be used. In
Her preexercise albuterol should not be “counted” addition, the patient should contact the clinician for
when assessing control; only the use of a SABA to treat follow-up instructions; the patient could continue the
symptoms is counted. SABA every 3–4 hours regularly for 24–48 hours. An
OCS burst could be considered, but this is usually un-
5. Answer: C necessary for a mild exacerbation.
This patient has moderate persistent asthma. Although
he has asthma symptoms only during the day once or 9. Answer: C
twice weekly, he has nighttime symptoms twice week- In an asthma action plan, the red zone instructions are
ly. One of the criteria for the moderate persistent cat- albuterol HFA 2–6 puffs; repeat in 20 minutes; start
egories is nighttime awakenings for symptoms more OCS burst; and then reassess. For red zone exacerba-
than once weekly but not nightly. tions, the higher dose (e.g., 6 puffs) is usually recom-
mended. If the response is incomplete or poor, repeat
6. Answer: D a high dose of a SABA. If there is no response, go to
This patient requires step 3 therapy for moderate per- the ED or call 911. Of note, the patient should call 911
sistent asthma. Medium-dose ICS alone is preferred immediately if he or she cannot walk or talk because of

ACCP Updates in Therapeutics® 2012: The Pharmacotherapy Preparatory Review and Recertification Course

1-277
Ambulatory Care

shortness of breath or if lips or fingernails are blue. The (on continuous oxygen therapy or using systemic cor-
treatment can begin while waiting for the ambulance to ticosteroids, in addition to a history of exacerbation
arrive. In addition, the patient should go to the hospital requiring an ED visit or hospitalization). This patient
if there is no improvement after 15 minutes of taking a does not meet these criteria.
high-dose SABA.
15. Answer: D
10. Answer: C According to the latest GOLD guidelines, oral corti-
Because he is experiencing shortness of breath at rest, costeroids are indicated in most exacerbations. The
has trouble with conversation because of shortness of recommended dose is oral prednisolone (or equivalent)
breath, and has an FEV1 less than 40%, his asthma ex- 30–40 mg/day for 10–14 days. Prednisone has a dose
acerbation is classified as severe. equivalent to prednisone. Antibiotic treatment is also
indicated because J.J. has all three cardinal symptoms
11. Answer: D of airway infection, which are 1) increased sputum pu-
For severe asthma exacerbations in the ED setting, the rulence, 2) increased sputum volume, and 3) increased
recommended treatment is oxygen to achieve an Sao2 of dyspnea. Trimethoprim/sulfamethoxazole is one of the
90% or greater, high-dose inhaled SABA PLUS ipratro- recommended antibiotics.
pium by either nebulizer or MDI with valved holding
chamber every 20 minutes for 1 hour or continuously, 16. Answer: A
and OCSs. For a DVT or pulmonary embolism of a known cause,
the recommended duration of warfarin is 3 months (1A
12. Answer: B recommendation).
S.H. is in GOLD guidelines patient group B because his
postbronchodilator FEV1 is between 50% and 80%, he 17. Answer: D
has had one or fewer exacerbations in the past year, and The goal INR is 2.5–3.5 for patients with mechanical
his mMRC score is 2 or greater. If the CAT score were valves in the mitral position. If the mechanical valve is
being used, the score would be 10 or greater. in the aortic position with no additional risk factors, the
goal is 2–3. The addition of aspirin (50–100 mg/day) is
13. Answer: B recommended with mechanical valves if bleeding risk
According to the GOLD guidelines, the recommended is low, as in M.H.
treatment for patient group B is regular treatment with
a long-acting bronchodilator (either LABA or LA anti- 18. Answer: C
cholinergic), in addition to a short-acting bronchodila- For aortic mechanical valves, the goal INR is 2–3 if the
tor as needed. Inhaled corticosteroids are recommended left atrium is of normal size and the patient is in normal
only in groups C and D. Daliresp is only recommended sinus rhythm.
if FEV1 is less than 50% of predicted with chronic bron-
chitis and a history of frequent exacerbations. 19. Answer: C
The INR increased because fluoxetine (S-isomer) was
14. Answer: A added. The omeprazole (R-isomer) interaction is more
The patient is in GOLD risk group B, on the basis of chronic, and the warfarin dose was likely adjusted 6
spirometry and CAT score, and is on the first-choice months ago when omeprazole was initiated. The 2012
therapy. Because his control is worsening, he should go CHEST guidelines suggest against routinely giving vi-
to the second-choice therapy, for which combined long- tamin K if the INR is 10 or less and there is no evidence
acting bronchodilators can be used. Inhaled corticoste- of bleeding. Warfarin should be held for 1–2 days; INR
roids are only recommended in risk groups C and D. should be monitored more often; and warfarin should be
Although a recent study showed benefits with chronic reinitiated at a lower dose when the INR approaches 3.
azithromycin, the guidelines do not recommend regu-
lar treatment with long-term antibiotics. In addition, 20. Answer: C
the study showing the benefit of azithromycin included Lowering the dose to 4 mg/day would be a 20% reduc-
only patients who were at higher risk of exacerbations tion in the weekly dose. In general, it is recommended

ACCP Updates in Therapeutics® 2012: The Pharmacotherapy Preparatory Review and Recertification Course

1-278
Ambulatory Care

to lower the dose by 5%–20% when the INR is high.


Because the INR is more than moderately elevated and
the drug interaction causing the high INR is continu-
ing, a 20% reduction in dose would be the most prudent
action. Answer A and Answer B are dose reductions
much greater than 20%, and Answer D is less than a
5% dose reduction.

21. Answer: C
For high-risk patients, therapeutic-dose LMWH is rec-
ommended for bridge therapy before surgery, while
holding warfarin. Enoxaparin 1 mg/kg 2 times/day is
the therapeutic dose; 30 mg 2 times/day is the prophy-
laxis dose (which is generally no longer recommended
for bridging therapy). He has atrial fibrillation and is at
high risk because his CHADS2 score is 5 (hypertension,
age 75 or greater, diabetes [1 point each], and history of
TIA [2 points]).

22. Answer: D
The CDC recommends that the influenza vaccine be
given every year in every individual 6 months and old-
er. People 65 years and older should have a one-time
pneumococcal revaccination if they were vaccinated
5 or more years previously and were younger than 65
years at the time of primary vaccination. Zoster vacci-
nation is recommended in all adults 60 years and older,
regardless of previous zoster infection. The Tdap vac-
cine is recommended in individuals, regardless of age, who
are close contacts of infants younger than 12 months, so
she should also receive Tdap because of close contact with
her 3-month-old grandson.

23. Answer: D
First-year college students up to age 21 if not previously
vaccinated on or after age 16 living in dormitories should
receive meningococcal vaccine (MCV4 – Menactra).
Pneumococcal polysaccharide vaccine is recommended
for smokers aged 19–64, and this patient is a smoker.
The Td vaccine should be given every 10 years. In adults
younger than 65, a one-time dose of Tdap should be given,
regardless of the time interval since the most recent tetanus
vaccination. Human papillomavirus vaccine is for girls and
women aged 11–26 years. Ideally, it should be given before
the start of sexual activity, but it should still be adminis-
tered to sexually active girls and women.

ACCP Updates in Therapeutics® 2012: The Pharmacotherapy Preparatory Review and Recertification Course

1-279
Ambulatory Care

Answers and Explanations to Self-Assessment Questions

1. Answer: D years since this patient’s last Td, which was given as
That she uses her inhaler throughout the day, everyday, Tdap. The HZV (Zostavax) is recommended at age
and sometimes at night indicates she has daily symptoms. 60 and older by the CDC; however, it is indicated
Severe persistent: frequency of symptoms is throughout at age 50 and older in the manufacturer’s package
the day; nighttime symptoms are often 7 times/week; a insert.
SABA several times a day; normal activity is extremely
limited; FEV1 less than 60% of predicted; FEV1/FVC 6. Answer: B
reduced more than 5%. The goal INR is 2.0–3.0.

2. Answer: C 7. Answer: C
“Severe persistent” initial treatment is step 4 or 5. Venous thromboembolism during pregnancy should
Step 4 preferred: inhaled steroid (medium dose) plus be treated with UFH or LMWH throughout pregnancy
a LABA. Alternative: inhaled steroid (medium dose) and then switched to warfarin after delivery (even if
plus either an LTM or sustained-release theophylline or breastfeeding). Anticoagulants should be continued for
zileuton. Step 5 preferred: inhaled steroid (high dose) at least 6 weeks postpartum for a minimum total dura-
plus a LABA. tion of 3 months.

3. Answer: D 8. Answer: B
Ratio data are ranked in a specific order with a consis- This patient is in GOLD patient group B. A single long-
tent level of magnitude difference between units; with acting bronchodilator is first choice for medication
an absolute zero. treatment. Tiotropium (Spiriva) is a long-acting bron-
chodilator (anticholinergic) that would be appropriate
4. Answer: B to initiate in this patient. A LABA would also be appro-
The percentage of patients receiving fluticasone/salme- priate, but it was not one of the choices. Zafirlukast and
terol who have an asthma-related hospitalization will omalizumab are recommended for asthma, not COPD.
be compared with the percentage of patients receiving An ICS is recommended only in patient group C or D.
fluticasone who have an asthma-related hospitalization.
We assume that these two groups are normally distrib-
uted. These data are considered nominal. Chi-square
test is appropriate to analyze nominal or categoric data.
Analysis of variance is appropriate when there are more
than two treatment groups. The Student unpaired t-test
is used for continuous data that are normally distrib-
uted. The Mann-Whitney U test is appropriate when
continuous data are not normally distributed.

5. Answer: B
Pneumococcal vaccine is recommended in indi-
viduals aged 19–64 with asthma. She falls into this
category. Influenza vaccine is recommended in in-
dividuals with chronic cardiovascular or pulmonary
diseases such as asthma. However, usually, the in-
fluenza vaccine is given in the fall or early winter to
offer protection when the risk of infection is highest.
The tetanus booster (Td) (tetanus and diphtheria) is
recommended every 10 years, and it has not been 10

ACCP Updates in Therapeutics® 2012: The Pharmacotherapy Preparatory Review and Recertification Course

1-280

S-ar putea să vă placă și